RR - Cardiology 2

Lakukan tugas rumah & ujian kamu dengan baik sekarang menggunakan Quizwiz!

Lower Limits of Pediatric Systolic Blood Pressure

0-28 days: 60 mm Hg • 1-12 months: 70 mm Hg • 1-10 years: 70 mm Hg + (2 x age in years) • >10 years: 90 mm Hg

Why do H2 blockers help with cutaneous symptoms in allergic reactions?

10% of histamine receptors in the skin are of the H2 variety.

What is the normal range for the PR interval?

120-200 milliseconds.

What is the recommended blood pressure goal in patients with hypertension?

139/89 mm Hg or less.

Which of the following patients should receive prophylactic antibiotics to prevent endocarditis? 18-year-old pregnant woman with a history of a repaired congenital heart defect, with an impending vaginal delivery 19-year-old woman with a history of endocarditis who is undergoing a dental extraction 20-year-old man with a prosthetic heart valve who requires a Foley catheter due to urinary obstruction 21-year-old man with a history of a heart transplant and valvulopathy who is undergoing suture repair of a facial laceration

19-year-old woman with a history of endocarditis who is undergoing a dental extraction This patient meets high-risk criteria (history of endocarditis) and requires antibiotic prophylaxis. The American Heart Association has published guidelines regarding prophylaxis for infective endocarditis in high-risk patients undergoing dental or invasive respiratory procedures. High-risk patients who are undergoing vaginal delivery (A), Foley catheterization (C), and suture repair (D) do not require antibiotic prophylaxis. In fact, prophylactic antibiotics are generally not needed with any genitourinary or gastrointestinal instrumentation and nearly all procedures that are performed in the ED.

A patient with an intermediate risk of coronary artery disease is undergoing an exercise stress test. Which of the following is the most specific finding for myocardial ischemia? 2 mm downsloping ST-segment depression 2 mm upsloping ST-segment depression Increase of systolic blood pressure Sporadic premature ventricular complexes

2 mm downsloping ST-segment depression 2 mm downsloping ST-segment depression is the most specific finding for myocardial ischemia during an exercise stress test. Subendocardial ischemia during exercise produces ST-segment depression or elevation or both. ST-segment depression that occurs during exercise testing is one of the most identifiable ECG signs of myocardial ischemia. The ECG portion of the exercise test is generally considered abnormal, or positive for ischemia, when there is ≥ 1 mm horizontal or downsloping ST-segment depression in one or more leads. Horizontal or downsloping ST-segment depression is generally more specific for ischemia than upsloping ST-segment depression. An increase in systolic blood pressure (C) is an expected physiologic response during an exercise stress test and is not a sign of myocardial ischemia. Sporadic premature ventricular complexes (D) are not uncommon during peak exercise and do not indicate myocardial ischemia. Although 2 mm upsloping ST-segment depression (B) could be suggestive of myocardial ischemia, it is less specific than downsloping or horizontal ST-segment depression.

How long should an individual undergo treatment for an isolated pulmonary embolism with no other risk factors?

3 months.

What is the desired heart rate in the treatment of chronic stable angina with beta-blocker therapy?

55-60/min.

What percent of aortic aneurysms >5 cm can be palpated?

75%, but only 5-10% of patients with an AAA have an abdominal bruit.

What ankle-brachial index measurement is frequently seen with resting pain?

< 0.4

What duration of CPR constitutes a relative contraindication to fibrinolytic use for STEMI?

> 10 minutes.

Ventricular Tachycardia

>3 consecutive ectopic ventricular beats • Monomorphic, polymorphic • Bidirectional: digoxin toxicity • Wide complexes • Pulseless: immediate defibrillation • Unstable: synchronized cardioversion • Stable: procainamide, amiodarone, synchronized cardioversion (refractory) • If unsure, manage all wide complex tachycardias as ventricular tachycardia

Which of the following patients is considered hypotensive? A 1-year-old boy with a systolic blood pressure of 75 mm Hg A 2-year-old girl with a systolic blood pressure of 80 mm Hg A 3-year-old girl with a systolic blood pressure of 70 mm Hg A 6-year-old boy with a systolic blood pressure of 85 mm Hg

A 3-year-old girl with a systolic blood pressure of 70 mm Hg A 3-year-old girl with a systolic blood pressure of 70 mm Hg is considered to be hypotensive. The minimum systolic blood pressure in a child (1-10 years) is calculated by the formula: Minimum SBP = 70 + (2 x age in years). Therefore, the minimum systolic blood pressure for a 3-year-old is 76 mm Hg. The minimum systolic blood pressure for a neonate (0-28 days) is 60 mm Hg and an infant (1-12 months) is 70 mm Hg. Based on the formula, a systolic blood pressure of 75 mm Hg is considered normal for a 1-year-old (A). A systolic blood pressure of 80 mm Hg is normal for 2-year-old (B) and a systolic blood pressure of 85 mm Hg for a 6-year-old (D).

Which of the following patients requires endocarditis prophylaxis for a dental procedure that requires manipulation of the gingival tissue? A 14-year-old boy with a Still's murmur A 25-year-old man with a grade 3/6 systolic ejection murmur A 35-year-old man with a prosthetic mitral valve A 45-year-old man with aortic stenosis

A 35-year-old man with a prosthetic mitral valve To prevent adverse complications from infective endocarditis, recent guidelines indicate that only high-risk cardiac patients should receive bacterial endocarditis prophylaxis when undergoing dental procedures that involve manipulation of gingival tissue or periapical teeth or perforation of oral mucosa. The routine use of infective endocarditis prophylaxis before gastrointestinal or genitourinary tract manipulation is not recommended, except in patients with active GI or GU infections.The high-risk cardiac patients include those with prosthetic cardiac valves or prosthetic material used for cardiac valve repair, prior history of infective endocarditis, unrepaired congenital heart disease, or repaired congenital heart defect with a prosthesis during the first 6 months after the procedure. Heart murmurs do not require prophylaxis for dental procedures unless they fall in high-risk category described above. Therefore a 3/6 systolic ejection murmur (B), Still's murmur (A) and aortic stenosis (D), do not require antibiotic prophylaxis.

Which of the following clinical scenarios can be defined as a hypertensive emergency? A 25-year-old pregnant woman in her second trimester with a blood pressure of 155/100 mm Hg with a normal urinalysis A 55-year-old man with a blood pressure of 185/90 mm Hg whose creatinine has increased from 1.0 to 2.5 mg/dL within 36 hours A 59-year-old asymptomatic man requesting a medication refill and is found to have a blood pressure of 210/110 mm Hg and an ECG consistent with left ventricular hypertrophy (LVH) A 63-year-old woman with a history of poorly controlled hypertension who presents with a finger laceration and is noted to have a blood pressure of 200/105 mm Hg

A 55-year-old man with a blood pressure of 185/90 mm Hg whose creatinine has increased from 1.0 to 2.5 mg/dL within 36 hours Hypertensive emergency is generally defined as a markedly elevated blood pressure in the setting of acute end-organ damage of the cardiovascular, neurologic, or renal organ system. This condition is a true medical emergency and warrants early reduction of blood pressure (preferably within one hour of identification of the condition) with titratable intravenous medications. It is important to understand, however, that an elevated blood pressure in response to an acute condition is often physiologic; aggressive lowering of the pressure in these conditions (e.g., ischemic stroke) may actually increase morbidity and mortality. Renal failure can be seen as both a consequence and cause of hypertension. Uncontrolled hypertension may cause acute kidney injury and can accelerate the progression of injury in patients with chronic renal failure. Acute worsening of kidney function as seen in this patient whose creatinine increased acutely from 1 to 2.5 mg/dL—in the setting of elevated blood pressure—should be considered a hypertensive emergency and warrants immediate treatment. Hypertension is one of the most common complications in pregnancy. Hypertension in pregnancy is defined as a systolic pressure > 140 mm Hg or a diastolic pressure > 90 mm Hg. Severe hypertension is classified as a systolic pressure > 160 mm Hg or diastolic > 105 mm Hg. Pre-eclampsia is defined as hypertension occurring after 20 weeks gestation with proteinuria or any signs or symptoms of end-organ damage (e.g., elevated LFTs) and should be considered a hypertensive emergency. Hypertension occurring after 20 weeks gestation without these signs or symptoms (A) is termed gestational hypertension. Patients who are hypertensive, but asymptomatic, and show no evidence of acute end-organ damage (C and D) in most cases, do not need acute lowering of their blood pressure. Patients with chronic hypertension may have an altered autoregulatory range; rapid normalization of their elevated blood pressure may in fact lead to hypoperfusion and ischemia. In response to chronic hypertension, the heart is remodeled in a cycle starting with increased wall stress that leads to hypertrophy and impaired diastolic function. When LVH is present in the setting of hypertension, early follow-up should be arranged because such patients are at increased risk for MI, heart failure, stroke, and sudden death.

A diagnosis of orthostatic hypotension can be made in which of the following scenarios? A drop in diastolic blood pressure of 5 mm Hg upon standing A drop in systolic blood pressure of 20 mm Hg upon standing An increase in heart rate of 35 beats per minute upon standing An increase in heart rate of 5 beats per minute upon standing

A drop in systolic blood pressure of 20 mm Hg upon standing Orthostatic hypotension is defined as a decrease in systolic BP of 20 mm Hg or diastolic BP of 10 mm Hg within 3 minutes of standing from a seated or supine position. Symptoms may include positional syncope, dizziness, generalized weakness or fatigue. Once underlying cardiac, endocrine, neurologic and medication side effect etiologies are ruled out, treatment options include increasing water intake, sodium supplementation, abdominal or lower extremity binders, a physical therapy conditioning program and medications. A diagnosis of orthostatic hypotension requires a drop of 10 or more mm Hg (A) in diastolic BP upon standing. Although commonly checked along with blood pressure measurements during supine and standing position, baseline heart rate, or changes in heart rate (C), are not included in the latest definition of orthostatic hypotension. Postural orthostatic tachycardia syndrome, not orthostatic hypotension, should be considered if the heart rate increases ≥ 30 beats per minute (D) upon standing.

What is the classic physical sign associated with tricuspid regurgitation?

A large, bounding v wave is seen during jugular vein inspection. Also, a pulsatile liver may be palpable.

In which of the following clinical scenarios is an implantable cardioverter-defibrillator indicated for the prevention of ventricular dysrhythmias and sudden cardiac death? A patient with a left ventricular ejection fraction < 35% and heart failure NYHA Functional Class II or III A patient with a normal left ventricular ejection fraction and asymptomatic structural heart disease A patient with sustained ventricular tachycardia in the setting of an acute myocardial infarction A patient with sustained ventricular tachycardia in the setting of hyperkalemia

A patient with a left ventricular ejection fraction ≤ 35% and heart failure NYHA Functional Class II or III A patient with a left ventricular ejection fraction ≤ 35% and heart failure NYHA Functional Class II or III is a clinical scenario in which an implantable cardioverter-defibrillator is indicated. An implantable cardioverter-defibrillator is a small device combining a cardioverter and defibrillator into one implantable unit that is surgically placed in the chest or abdomen. It is battery powered and programmed to detect dysrhythmias, mainly sustained ventricular tachycardia and ventricular fibrillation, which can lead to sudden cardiac death. It has a very high success rate in rapidly terminating ventricular dysrhythmias and evidence shows that it improves survival. Implantable cardioverter-defibrillator implantation is generally considered the first-line treatment for the secondary prevention of sudden cardiac death in patients who have survived an event and for primary prevention in certain high risk populations. Published guidelines exclude cases that are considered "reversible causes." Some of the major indications are as follows. A patient with asymptomatic structural heart disease (B) is not an indication for implantable cardioverter-defibrillator. Only when a patient with structural heart disease, such as hypertrophic cardiomyopathy, has symptoms or unexplained syncope with inducible dysrhythmia on electrophysiological studies is it indicated for an implantable cardioverter-defibrillator. Patients with sustained ventricular tachycardia in the setting of hyperkalemia (D) or acute myocardial infarction (C) are not indicated for an implantable cardioverter-defibrillator. Hyperkalemia and acute ischemia are considered "reversible causes" and correction of the metabolic derangement and revascularization are often adequate measures to reduce the risk of sudden cardiac death.

What are Osler's nodes?

A sign of subacute bacterial endocarditis, these are the result of immune complex deposition, leading to tender nodules in the digit pads.

Which of the following conditions is most suggestive of an asymptomatic abdominal aortic aneurysm? Abdominal bruit Abdominal mass Hypertension Hypotension

Abdominal mass An abdominal mass is most suggestive of an asymptomatic abdominal aortic aneurysm. An abdominal aortic aneurysm is defined as a focal, full-thickness dilation of the aorta that is greater than 50% larger than its normal diameter. Most patients with abdominal aortic aneurysms do not have any symptoms. Asymptomatic abdominal aortic aneurysms may be discovered as a result of screening in patients with risk factors for abdominal aortic aneurysm, on routine physical examination, or on imaging studies to evaluate an unrelated complaint. An abdominal aortic aneurysm does not typically cause symptoms unless the aneurysm is expanding rapidly, has become large enough to compress surrounding structures, is an inflammatory or infectious aneurysm, or has ruptured. Patients with symptomatic abdominal aortic aneurysms most commonly present with abdominal, back, or flank pain, which may or may not be associated with abdominal aortic aneurysm rupture. The classic triad of severe acute pain, a pulsatile abdominal mass, and hypotension occurs in about 50% of patients with ruptured abdominal aortic aneurysm. An abdominal bruit (A) is not most suggestive of an asymptomatic abdominal aortic aneurysm, but is more suggestive of renal artery stenosis. Hypertension (C) is not most suggestive of an asymptomatic abdominal aortic aneurysm. Hypertension is a risk factor for development of an aortic aneurysm. Hypotension (D) is not most suggestive of an asymptomatic abdominal aortic aneurysm but could result from a ruptured abdominal aortic aneurysm with hypovolemic shock.

What is the classic triad of AAA?

Abdominal/back pain, hypotension and a pulsatile abdominal mass.

What is the preferred therapy for the prevention of recurrent preexcited atrial fibrillation?

Ablation of the accessory pathway.

Aortic Regurgitation

Acute MCC: endocarditis • Chronic MCC: rheumatic heart disease • Blowing diastolic murmur at left sternal border • Pulse pressure: normal (acute), widened (chronic) • de Musset sign: head bobbing with systole • Quincke's pulse: prominent nail pulsations • Duroziez murmur: "singsong" murmur over femoral artery • Austin-Flint murmur: mid-diastolic murmur in severe AR • Rx objective: ↓ afterload

A 54-year-old man presents to the hospital in acute respiratory distress. He was released from the hospital three days ago after undergoing a cardiac stent placement secondary to an acute myocardial infarction. His hospital course was uncomplicated. On exam, his BP is 110/60 mm Hg, HR is 115, RR is 28, and pulse oximetry is 91% on room air. Cardiopulmonary exam reveals a midsystolic murmur with bibasilar crackles. An ECG shows sinus tachycardia. Which of the following is the most likely diagnosis? Acute aortic insufficiency Acute mitral regurgitation Postmyocardial infarction syndrome Ventricular aneurysm rupture

Acute mitral regurgitation Acute mitral regurgitation is the result of rupture of the chordae tendineae, papillary muscle, or valve leaflet. Its etiology can be idiopathic due to acute ischemia, a complication of infective endocarditis, or trauma. As with this patient, it may also occur as a delayed consequence of acute myocardial infarction (usually two to seven days post-event). Patients typically present in fulminant pulmonary edema rapid in onset. It is associated with a midsystolic murmur. Patients generally have no history of heart failure, and the ECG may be normal or display signs of ischemia or infarction. Acute aortic insufficiency (A) is most commonly due to infective endocarditis, aortic dissection, or trauma. As in acute mitral regurgitation, patients present with severe dyspnea due to a rapid rise in left ventricular end-diastolic pressure leading to pulmonary edema and cardiogenic shock. Notable findings on physical exam include signs of decreased cardiac output (cool, pale extremities, cyanosis), a loud S3, and a short diastolic murmur best heard at the left sternal border with the diaphragm of the stethoscope. Postmyocardial infarction syndrome (C) is also known as Dressler's syndrome and is associated with pericarditis that occurs in the setting of injury to the heart or pericardium. Patients typically present with fever, pleuritic chest pain, and a pericardial effusion. While left ventricular aneurysm (D) may result from an acute myocardial infarction, rupture is rare because the aneurysm wall is lined by scar tissue. However, when present, it may block the aortic outflow tract leading to reduced cardiac output.

A 42-year-old man presents to the Emergency Department with acute onset of severe shortness of breath. He was recently discharged from the hospital with endocarditis. Vital signs include temperature 37.6oC, HR 110, BP 110/50, RR 24, Pox 93%. On examination, he is ill appearing with a new harsh apical systolic murmur and bibasilar rales. Which of the following is the most likely diagnosis? Acute mitral regurgitation Aortic regurgitation Health care associated pneumonia Pulmonary embolism

Acute mitral regurgitation Acute mitral regurgitation presents with acute onset of severe dyspnea and signs of pulmonary edema. Patients can rapidly decompensate into cardiogenic shock or cardiac arrest. A harsh systolic apical murmur and S4 gallop are present. It is most commonly due to rupture of the papillary muscle or chordae tendinae secondary to myocardial infarction. Other causes include rupture due to infective endocarditis, blunt chest trauma or spontaneous rupture. Chest x-ray will show signs of pulmonary edema with a normal heart size (unless the patient has chronic ischemic or valvular heart disease). Echocardiography will confirm the diagnosis and assess the degree of regurgitation. Emergency Department management includes oxygen and positive pressure ventilation as needed for respiratory failure. Nitrates and diuretics are indicated for treatment of pulmonary edema. An intra-aortic balloon pump may be needed for stabilization of the hypotensive patient until definitive treatment with surgery. Emergent cardiology and surgical consultation is imperative. Aortic regurgitation (B) can also present with severe dyspnea but is associated with a high pitched blowing diastolic murmur. Health care associated pneumonia (C) typically presents with fever, cough and findings of consolidation on lung exam. Although dyspnea may be severe and acute in onset in patients with pulmonary embolism (D), they do not typically present with a new heart murmur.

A 34-year-old woman presents to the ED with chest pain that is worse with inspiration and better upon leaning forward. She has had a runny nose and cough for the last week. In the ED, her vital signs are BP 134/78, HR 86, RR 14, oxygen saturation 99% on room air, and T101°F. On exam, a friction rub is heard. An ECG displays global ST segment elevation with PR segment depression. What is the most likely diagnosis and what would be the next step in management? Acute myocardial infarction; give aspirin, nitroglycerin, consult cardiology, and activate the cath lab Acute pericarditis; give nonsteroidal anti-inflammatory drugs Cardiac tamponade; perform immediate pericardiocentesis Pulmonary embolism; order CT angiography of the chest

Acute pericarditis; give nonsteroidal anti-inflammatory drugs This patient most likely has acute pericarditis, which is inflammation of the pericardial sac. Patients present with pleuritic chest pain that is typically worse when lying supine, deep inspiration, or swallowing. The pain is usually relieved by leaning forward. On auscultation, a pericardial friction rub may be heard. Pulsus paradoxus may also be observed, which is a fall in systolic blood pressure of greater than 10 mmHg with inspiration. Pericarditis can have many etiologies including infection, systemic connective tissue diseases, uremia, post-radiation, or post-myocardial infarction (Dressler's syndrome). Although there is no definitive diagnostic test, an ECG can demonstrate diffuse ST segment elevation, diffuse PR segment depression, and PR elevation in aVR (thumbprint sign). Treatment of pericarditis is mainly supportive. NSAIDs will reduce inflammation and pain. Steroids or colchicine may be given for refractory cases. An acute myocardial infarction (A) is less likely in this patient given her age and clinical presentation (fever, runny nose, and cough). A pericardial effusion may accompany acute pericarditis but rarely leads to cardiac tamponade (C). The clinical triad of pericardial tamponade is hypotension, jugular venous distension, and muffled heart sounds. A pulmonary embolism (D) is less likely in this patient given her ECG findings, vital signs, and clinical presentation.

A 70-year-old woman with a long history of coronary artery disease is seen in cardiology clinic for routine follow up. She complains of continued angina despite medical therapy. She reports chest pain every time she walks to her mailbox. It does not occur at rest and is relieved by sublingual nitroglycerin. Her medications include carvedilol, amlodipine, daytime transdermal nitroglycerine, sublingual nitroglycerin, aspirin, and simvastatin. On physical exam her BP is 105/72, HR is 51 and RR 16. What is the next step in management? Add ranolazine Increase the dose of carvedilol Increase the dose of simvastatin Schedule 24 hour usage of transdermal nitroglycerin

Add ranolazine The addition of ranolazine should be considered in patients with chronic stable angina who remain symptomatic despite optimal doses of beta blockers, calcium channel blockers and nitrates. Stable angina refers to chest discomfort that occurs predictably and reproducibly at certain levels of exertion and is relieved by rest or nitroglycerine. This patient remains symptomatic on carvedilol, amlodipine and two different preparations of nitroglycerine. Ranolazine is a novel anti-anginal agent that causes selective inhibition of the late sodium channel. It is approved for the treatment for chronic stable angina for those who have failed standard medical therapy. It has been shown to be effective in reducing anginal symptoms and improving exercise capacity when added to conventional medical therapy. Ranolazine can prolong the QT interval and it should be used with caution in patients with kidney or liver disease. It is not safe to increase the dose of carvedilol (B) as this patient's blood pressure and pulse are too low for up-titration of her beta-blocker. Statin therapy is important for the treatment of this patient's coronary artery disease; however it is not appropriate to increase the dose of simvastatin (C) in this situation as statin therapy does not have anti-anginal properties. Long acting nitroglycerin preparations are very useful in the treatment of chronic stable angina; however, they require a nitrate free period of 8-12 hours, typically at night, to obviate nitrate tolerance. Therefore, 24 hour use of transdermal nitroglycerin (D) would not be beneficial.

Why should beta-blockers be avoided in the treatment of patients with cocaine related chest pain or hypertension?

Administration of a beta-blocker can cause unopposed alpha effects leading to worsening symptoms and blood pressure.

Abdominal Aortic Aneurysm (AAA)

Advanced age, male, smoking hx, HTN • Acute abdominal pain + hypotension + pulsatile abdominal mass • US: 100% sensitive • CT: 100% sensitive, detects rupture/leak • AAA > 5 cm: ↑ risk of rupture • Renal colic in elderly: r/o AAA

What antibiotic is recommended as prophylaxis for high-risk patients undergoing a dental extraction?

Amoxicillin (2 grams by mouth).

What is the most common cause of restrictive cardiomyopathy?

Amyloidosis.

In patients with aortic dissection, what is the recommended blood pressure goal?

Antihypertensives should be titrated to a systolic blood pressure of 110 mm Hg.

List some medications whose side effects can cause symptoms of orthostasis?

Antiparkinsonian drugs, antiadrenergics, anticholinergics, antidepressants, antiarrhythmics, antipsychotics, diuretics, narcotics and sedatives.

Antithrombotics include which medications?

Antiplatelets (aspirin, glycoprotein IIb/IIIa, adenosine diphosphate, cyclooxygenase, phosphodiesterase, and thromboxane inhibitors), anticoagulants (vitamin K, factor Xa and thrombin inhibitors) and thrombo-fibrinolytics (plasminogen activators).

Who should be screened for an abdominal aortic aneurysm?

Any man between the ages of 65 and 75 years who has ever smoked.

A careful cardiac examination requires close attention to the heart sounds. The second heart sound, S2, is produced by which of the following structures? Aortic and pulmonic valves Mitral and tricuspid valves Pericardium and chest wall Posterior and anterior cusps of the mitral valve

Aortic and pulmonic valves The cardiac cycle begins with ventricular contraction, or systole. The pressure generated closes the mitral and tricuspid valves, producing S1, and opens the aortic and pulmonic valves. As the ventricles relax, the previously "pumped-out" blood pushes back on the heart, closing the aortic and pulmonic valves and producing the second heart sound, S2. The ECG's P wave comes before S1, the QRS complex occurs during S1 and the T wave occurs between S1 and S2. S1 is produced by the closure of the mitral and tricuspid valves (B). A friction rub may be appreciated when there is fluid or irritation between the pericardium and the chest wall (C). Functional abnormalities of the mitral valve's posterior and anterior cusp (D) may lead to a stenotic or regurgitant murmur.

Other than cardiac ischemia or infarct, what are some other cardiac causes of chest pain?

Aortic dissection, pericarditis and myopericarditis.

Which of the following statements is true regarding giant cell arteritis? Aortic involvement can lead to valvular disease and dissection Corticosteroid therapy should be initiated only when biopsy confirms the disease Histologic findings of inflammation are irreversible It is associated with sudden, painful binocular vision loss

Aortic involvement can lead to valvular disease and dissection Temporal arteritis is a chronic segmental vasculitis of medium and large vessels. Although it most commonly affects one or more branches of the carotid artery (temporal artery, ophthalmic artery, and posterior ciliary artery), the aorta can also be involved. Aortic involvement can lead to valvular insufficiency, aortic arch syndrome, and dissection. The carotid and vertebrobasilar arteries can also be affected, which can lead to neurologic complications. The condition is associated with a markedly elevated erythrocyte sedimentation rate (50-100 mm/hr). Histologic findings (C) are rapidly reversed with steroid therapy. Temporal arteritis is a sight-threatening disease, but timely administration of high-dose corticosteroids can prevent blindness. For this reason, if the diagnosis is suspected, corticosteroids should be administered immediately (B) while awaiting the results of temporal artery biopsy. Symptoms suggestive of temporal arteritis include headache, jaw claudication, and visual disturbances. Temporal arteritis is associated with a sudden painless monocular loss of vision (D) due to vascular occlusion of the ophthalmic or posterior ciliary artery with infarction of the optic nerve or retina.

Which of the following valvular disorders is characterized by any of the following: a low-pitched diastolic murmur heard best over the apex, an early high-pitched, blowing diastolic murmur heard best over the left sternal border, and a wide pulse pressure? Aortic regurgitation Aortic stenosis Mitral stenosis Tricuspid stenosis

Aortic regurgitation Physical signs of aortic regurgitation (AR) include a rapid, quick arterial pulse (Corrigan's pulse), a wide pulse pressure, an early high-pitched, blowing diastolic murmur heard best over the left sternal border, an S3 gallop, and a low-pitched diastolic murmur at the apex (Austin-Flint murmur). Aortic valve regurgitation is defined as blood flow from the aorta to the left ventricle in diastole because of an incompetent aortic valve. Aortic valve insufficiency is generally acquired through valve infection, dilation and dissection of the aortic root, trauma, or long-term degenerative change of the valve, particularly in the setting of hypertension. Patients with a history of prosthetic valves can also have aortic valve insufficiency. Aortic insufficiency can also be caused by a congenital bicuspid aortic valve. The surgical treatment of AR is indicated in symptomatic patients with dyspnea, angina, or CHF. Asymptomatic patients should undergo surgery if left ventricular ejection fraction is 55% or less, or left ventricular end-systolic dimension approaches 5.5 cm. Patients with moderate to severe AR should avoid competitive sports, heavy workloads, and weightlifting. The typical physical signs of severe aortic valve stenosis (B) are diminished carotid pulses (delayed and weak), a sustained apical impulse, a single second heart sound, an S4 gallop, and mid-systolic crescendo-decrescendo murmur late peaking best heard at the base of the heart. Most auscultatory signs of mitral stenosis (C) are missed if not performed in the left lateral decubitus position. Typically, the first heart sound (S1) is accentuated. A low-pitched diastolic rumble heard with the bell of the stethoscope over the apex is also present. The high-pitched opening snap, caused by the abrupt stopping of the domed mitral valve into the left ventricle is also appreciated in most patients midway between the left sternal border and apex. Tricuspid valve stenosis (D) is mostly caused by rheumatic heart disease and is typically associated with other valvular involvement. Patients can be dyspneic with activity. Typically, there is an increase in the jugular vein with a large a wave, indicating atrial contraction against a stiff tricuspid valve.

A 75-year-old otherwise healthy woman states that she has passed out three times in the last month during her daily brisk walk. Which one of the following is the most likely cause of her syncope? Aortic stenosis Atrial myxoma Orthostatic hypotension Vasovagal syncope

Aortic stenosis Syncope with exercise is a manifestation of organic heart disease in which cardiac output is fixed and does not rise with exertion. Syncope, commonly occurring with exertion, is reported in up to 42% of patients with severe aortic stenosis. The pathology of aortic stenosis includes processes similar to those in atherosclerosis, including lipid accumulation, inflammation, and calcification. The development of significant aortic stenosis tends to occur earlier in those with congenital bicuspid aortic valves. During the asymptomatic latent period, left ventricular hypertrophy and atrial enlargement of preload compensate for the increase in afterload caused by aortic stenosis. As the disease worsens, these compensatory mechanisms fail, leading to symptoms of heart failure, angina, or syncope. Doppler echocardiography is the recommended initial test for patients with classic symptoms of aortic stenosis. It is helpful for estimating aortic valve area, peak and mean transvalvular gradients, and maximum aortic velocity. Aortic valve replacement should be recommended in most patients with any of these symptoms accompanied by evidence of significant aortic stenosis on echocardiography. Vasovagal syncope (D) is associated with unpleasant stimuli or physiologic conditions, including sights, sounds, smells, sudden pain, sustained upright posture, heat, hunger, and acute blood loss. Orthostatic hypotension (C) is associated with changing from a sitting or lying position to an upright position. Atrial myxoma (B) is associated with syncope related to changes in position, such as bending, lying down from a seated position, or turning over in bed.

A 62-year-old man with a history of ongoing tobacco abuse, hypertension, dyslipidemia and erectile dysfunction complains of progressive aching pain in his right buttock and hip. The pain is worse with walking and is relieved with rest. Physical exam of the lower extremities reveals slightly diminished femoral, popliteal, and dorsalis pedis pulses. Which of the following is the most likely site of this patient's peripheral arterial disease? Aortoiliac artery Common femoral artery Popliteal artery Superficial femoral artery

Aortoiliac artery This patient most likely has peripheral arterial disease in the right aortoiliac artery. Peripheral arterial disease is physiologically significant atherosclerosis of the aortic bifurcations or arteries of the lower limbs. It is strongly associated with smoking, diabetes mellitus, and aging and shares all the risk factors common to atherosclerosis. This patient presents with right hip and buttock claudication, diminished femoral pulses and erectile dysfunction. This presentation commonly represents atherosclerotic disease within the aortoiliac system and is sometimes referred to as Leriche syndrome. Classic claudication is characterized by leg pain that is consistently reproduced with exercise and relieved with rest. The degree of symptoms of claudication depends upon the severity of stenosis, the collateral circulation, and the vigor of exercise. Patients with claudication can present with buttock, hip, thigh, calf, or foot pain, alone or in combination. The usual relationships are between pain location and corresponding anatomic site of arterial occlusive disease. Peripheral arterial disease in the common femoral artery (B) may cause thigh pain with effort but would not result in erectile dysfunction. Peripheral arterial disease in the popliteal artery (C) would produce pain in the lower one-third of the calf. Peripheral arterial disease within the superficial femoral artery (D) usually produces an effort-related discomfort in the upper two-thirds of the calf.

Hypertrophic Cardiomyopathy

Asymmetric LV septal wall hypertrophy → outflow obstruction • Autosomal dominant (familial form) • Young patient • Exertional syncope • Sudden cardiac death • S4 gallop • Midsystolic murmur (↑ as preload ↓) • Rx: ßBs or CCBs

Ethanol Intoxication

Ataxia, gait instability, slurred speech • Severe intoxication: 4 Hs (Hypotension, Hypoventilation, Hypothermia, Hypoglycemia) • Usually supportive care • Severe case rx: glucose, thiamine

Which of the following is an important predisposing factor for the development of the condition seen in this radiograph? Atherosclerosis Hernia Hyperparathyroidism Ulcer disease

Atherosclerosis The calcified wall of an abdominal aortic aneurysm (AAA) is visualized in this radiograph. The most common plain-film findings of an AAA is a curvilinear calcification of the aortic wall or a paravertebral soft tissue mass. Rarely with longstanding aneurysms, is the erosion of one or more vertebral bodies seen. Atherosclerosis, age >60 years, smoking, and family history are all important predisposing factors for the development of AAA. An AAA is a disease of aging and is rare before age 50 years. It is found in 5%-10% of elderly men screened with ultrasound. AAAs progressively enlarge, weakening the vessel wall, and ultimately rupture resulting in fatal hemorrhage. The most important factor in determining the risk of rupture is the size of the aneurysm. The rupture risk increases dramatically with increased aneurysmal size, and most ruptured AAAs have diameters >5 cm. Rupture usually occurs in the retroperitoneum; even those who make it to the OR still have a mortality close to 50%. The classic triad of a ruptured AAA is pain, hypotension, and a pulsatile abdominal mass, although many patients have only 1 or 2 of these components. Pain usually localizes to the abdomen, back, or flank, and is sometimes tragically misdiagnosed as renal colic. Treatment involves hemodynamic support and definitive repair by a vascular surgeon. Hernias (B) are the second most common cause of bowel obstruction, preceded by adhesions. Air fluid levels and dilated loops of bowel are radiographic findings associated with bowel obstruction. Primary hyperparathyroidism (C) is associated with some patients with recurring renal calculi. Secondary hyperparathyroidism can lead to abnormal bone resorption and manifests as bone syndromes such as rickets, osteomalacia, and renal osteodystrophy. Duodenal and gastric ulcers (D) are complicated by perforations that can lead to the detection of free air in the abdominal cavity. Free air is generally seen under the diaphragm in an erect patient.

What dysrhythmia is most commonly associated with tricuspid regurgitation?

Atrial fibrillation (80% of patients with tricuspid regurgitation).

A 50-year-old woman presents to the Emergency Department following a syncopal episode. On physical exam she is diaphoretic, but alert and orientated. Her blood pressure is 100/50 mm Hg, heart rate 46 beats per minute, and respirations 12 per minute. Her rhythm strip is shown above. What is the most appropriate treatment for this patient's condition? Adenosine Amiodarone Amlodipine Atropine

Atropine Sinus bradycardia is a rhythm that originates from the sinoatrial (SA) node with a rate of less than 60 per minute. The SA node is the heart's normal pacemaker, and sinus bradycardia may be normal in some patients such as conditioned athletes. In these cases, patients will have a resting heart rate of less than 60 per minute, but will not exhibit any other symptoms. Other times, sinus bradycardia may be due to organic heart disease such as coronary artery disease, cardiomyopathy, or myocarditis, resulting in symptomatic sinus bradycardia. Therefore, treatment largely depends on whether the patient is symptomatic (unstable) or not. An unstable patient is anyone who shows signs of poor perfusion such as altered mental status, diaphoresis, dizziness or syncope. A stable patient may be monitored closely, with ECG and vital signs, but initial treatment for an unstable patient is atropine. Since this patient presents with both a syncopal episode and diaphoresis, she is unstable and atropine is the appropriate therapy. After an initial dose of atropine, infusions of epinephrine or dopamine may also be considered, and for more emergent cases transcutaneous pacing may be necessary. Adenosine (A) is the treatment for supraventricular tachycardia , not bradycardia, and so is not appropriate for this patient. Amiodarone (B) is used to treat dysrhythmias such as ventricular tachycardia, not a sinus rhythm, while amlodipine (C) is a calcium channel blocker that helps to control blood pressure and chest pain.

A 55-year-old man presents after a syncopal event. He states he just started a new blood pressure medication. His heart rate is 41 beats/minute and his blood pressure is 95/60 mm Hg. Electrocardiogram shows sinus bradycardia. Which of the following medications should be administered? Adenosine Atropine Diltiazem Procainamide

Atropine Sinus bradycardia refers to a discharge rate from the sinoatrial node of < 60 beats/minute. Sinus bradycardia can be a result of pathologic factors like hypoxia, hypothermia, cardiac ischemia or infarction, hypothyroidism, or increased intracranial pressure. Many medications also cause sinus bradycardia, including beta-blockers, calcium-channel blockers, digoxin, and opioids. Sinus bradycardia may also be a normal finding in well-conditioned young people, athletes, during sleep, or as a result of vagal stimulation. On electrocardiogram, sinus bradycardia is indistinguishable from sinus rhythm other than having a slower rate. Patients with sinus bradycardia may be asymptomatic or may complain of dizziness or lightheadedness. An especially slow rate may result in signs of hypoperfusion (e.g. hypotension, altered mental status, or ischemic chest pain). The treatment of sinus bradycardia depends on the underlying cause and the clinical effects. Underlying causes should be corrected. Unstable patients should be treated with atropine while transcutaneous pacing is initiated and arrangements for transvenous pacing are made. Infusions of dopamine or epinephrine are also indicated to increase the heart rate if atropine is ineffective. Glucagon is used to treat cardiotoxicity from beta-blocker or calcium channel blocker overdose. Adenosine (A) is an atrioventricular (AV) nodal blocker used in the treatment of supraventricular tachycardias. Diltiazem (C) is a calcium channel blocker used in the treatment of tachydysrhythmias. Procainamide (D) is an antiarrhythmic used in the treatment of tachydysrhythmias.

A woman presents for her annual examination. She is treated for stable angina with aspirin and a statin. She continues to have angina 3-4 times a week, but its character, intensity, frequency and duration is unchanged. She has no history of myocardial infarction, diabetes or pulmonary disease. Which of the following interventions is most appropriate at this time? Begin atenolol Begin lisinopril Begin nitroglycerin Continue current medications and follow-up in 1 month

Begin atenolol Chronic stable angina is properly managed with lifestyle modifications and statin, antihypertensive and antiplatelet therapies. Concerning antihypertensive therapy in patients with documented coronary artery disease and chronic stable angina, first-line therapy is accomplished with beta-blockers. Even if patients do not have hypertension, beta-blockade is recommended. Their β1 and β2 antagonism decreases myocardial demand by decreasing heart rate and myocardial contractility, as well as increasing diastolic filling time. Other antihypertensive agents include ACE-inhibitors, ARBS and calcium-channel blockers. ACE-inhibitors (B) are considered first-line if the patient has had an MI or currently has diabetes. They are considered second-line therapy in all other hypertensive patients once beta-blockade therapy has been established. Nitrates (C) may be considered in patients with continued stable angina if they have failed beta or calcium-channel antagonists. This patient has not been trialed on either. Proper management of stable angina includes three classes of medications. This patient is missing an antihypertensive agent, and should be started on one at today's visit (D).

What is an example of a drug that reduces cardiac contractility?

Beta blockers

A 58-year-old woman presents with progressive orthopnea and peripheral edema. She also gets "winded" when she climbs a full flight of stairs. Which of the following laboratory tests helps define a cardiac versus a pulmonary cause of dyspnea? Beta-2 microglobulin Beta-type natriuretic peptide Erythrocyte sedimentation rate Homovanillic acid

Beta-type natriuretic peptide Cardiomyopathy is defined as a group of diseases which involve the muscle or electrical system of the heart. There are several causes, most of which are genetic in nature. Other etiologies are related to infectious, autoimmune, inflammatory, infiltrative, toxic, electrolytic, endocrine, nutritional and radiation etiologies. There are four main types: dilated, hypertrophic, restrictive and arrhythmogenic-right-ventricular (fibro fatty infiltration of the right ventricle). Dilated cardiomyopathy (DCM) is the most common subtype. It is the third most common cause of cardiac failure, behind coronary artery disease and hypertension. Adult DCM is most commonly caused by hypertension and coronary artery disease, but also is caused by genetic and infectious etiologies. Patients usually present with symptoms of heart failure, such as peripheral and pulmonary edema, cough, orthopnea and dyspnea at rest, with exertion or of the paroxysmal-nocturnal type. Initial evaluation of a patient with these symptoms includes electrocardiography, echocardiography, chest radiography and baseline chemistries, namely Beta-type (Brain) natriuretic peptide (BNP). BNP is secreted by the cardiac myocytes in response to increased volume and filling pressures. Beta-2 microglobulin (A) is used to evaluate hematologic disorders like multiple myeloma, lymphoma and leukemia. It is also associated with multiple sclerosis and other CNS disorders, as well as renal tubular disorders. Erythrocyte sedimentation rate (C) is a marker of inflammation. It is not a reliable test in differentiating cardiac from pulmonary dysfunction. Homovanillic acid (D) is a biomarker of metabolic stress in the central nervous system, not the cardiac or pulmonary systems.

Which of the following is a marker of high ventricular filling pressures? Brain natriuretic peptide Creatine kinase-MB Creatinine Troponin

Brain natriuretic peptide Brain natriuretic peptide is a marker of high ventricular filling pressures. It is a natriuretic hormone that was initially identified in the brain but is also released from the heart, particularly the ventricles. It is released in response to volume expansion and increased wall stress in the ventricles. Increased plasma concentrations are found in heart failure in response to increased ventricular filling pressures from volume overload. Troponin (D) and creatine kinase-MB (B) are markers for cardiac muscle damage as in an acute myocardial infarction or ischemia. Troponin is the preferred marker for the diagnosis of myocardial injury for all diagnostic categories because of its increased specificity and better sensitivity compared to creatine kinase-MB. Creatinine (C) is a marker of renal function, not increased ventricular filling pressures.

What are the three primary causes of aortic stenosis?

Calcification of a congenitally abnormal valve, calcification of a normal valve, and rheumatic heart disease.

What is the medical treatment of symptomatic premature atrial contractions?

Calcium-channel blockers or beta-blockers.

What is the formula for cardiac output?

Cardiac output (Q) = stroke volume (SV) x heart rate (HR).

A patient with dyspnea and angina fails medication management of his symptoms with beta-blockers, ACE-inhibitors and calcium channel blockers. He undergoes complete cardiac evaluation which uncovers the presence of nonobstructive, end-stage hypertrophic cardiomyopathy. Which of the following is the most appropriate treatment at this point in time? Aggressive diuresis Cardiac transplantation Implantable intracardiac pacing Surgical myectomy

Cardiac transplantation The management of hypertrophic cardiomyopathy should follow heart failure treatment guidelines. This includes a careful use of diuretics, as many patients with HCM require higher filling pressures to maintain cardiac function. This is especially true if edema is not a main finding. Negative inotropes and negative chronotropes are also recommended, and include beta-blockers and calcium channel blockers. If pharmacotherapy is unsuccessful, further treatment is dictated by whether the hypertrophic cardiomyopathy is obstructive or nonobstructive. If obstructive physiology is detected, surgical options may be required, and include cardiac pacing, surgical myectomy or septal ablation. If nonobstructive physiology is found, cardiac transplant is usually the only viable solution. Diuresis (A) is cautioned in those with hypertrophic cardiomyopathy. Acute, aggressive diuresis may likely push the patient into further decompensation. Pacing (C) and myectomy (D) may be viable treatment options for obstructive HCM but not for nonobstructive HCM.

How does carotid massage affect second degree type I and type II heart block?

Carotid massage will typically worsen second degree type I block and improve second degree type II block.

Dilated Cardiomyopathy

Causes: idiopathic > viral myocarditis, Chagas disease • Echo: 4 dilated chambers (ventricles > atria) • Systolic pump failure → heart failure • S3, S4 • Peripheral embolization of mural thrombi • MC dysrhythmia: atrial fibrillation • ßBs, ACEIs, diuretics, digoxin, anticoagulation, AICD

Mitral Regurgitation (MR)

Causes: ischemic heart disease, endocarditis, MI, trauma Acute sx: dyspnea, pulmonary edema, cardiogenic edema Chronic sx: asymptomatic until CHF onset, S3 Acute rx: nitroprusside, dobutamine, intra-aortic balloon pump, emergency surgery Chronic rx: anticoagulation, CHF rx, valve replacement

Tricuspid Regurgitation

Causes: tricuspid ring stretching > pulmonary HTN, endocarditis, rheumatic heart disease • Pansystolic murmur at left sternal border • JVP: giant c-v wave • Atrial fibrillation

What is the most commonly seen symptom or sign in patients with acute aortic dissection? Aortic insufficiency murmur Chest pain Pulse deficit Syncope

Chest pain Chest pain is the most common symptom seen in patients with acute aortic dissection. Aortic dissection is an uncommon presentation but it represents a difficult and life-threatening diagnosis. Difficulty in diagnosing the disease stems from the myriad of presentations and manifestations the disease can assume. Approximately 73% of patients with acute aortic dissection will present complaining of chest pain. This symptom is more common in those patients with ascending dissections whereas back pain is more common in those with descending thoracic dissections. Some complaint of pain is seen in up to 96% of patients. The pain is classically described as ripping or tearing but only about 50% of patients will describe it in this way. Aortic insufficiency murmur (A) results from an ascending dissection that compromises the aortic valve but is only seen in about 32% of patients. Pulse deficit (C) is even less common (15%). Syncope (D) is seen in approximately 9% of aortic dissections.

An elderly man presents with 4 episodes of angina in the past 24 hours. His medical history includes diabetes and advanced COPD. Based on initial testing, you diagnose non-ST-elevation myocardial infarction. You are waiting for the cardiac team to admit him to the critical care unit. In the interim, which of the following is the most appropriate medication to begin? Atelplase Clopidogrel Digoxin Metoprolol

Clopidogrel Non-ST-elevation myocardial infarction (NSTEMI) treatment begins with a basic anti-ischemic regimen consisting of oxygen, morphine, nitrates, and possibly beta-blockers and ACE-inhibitors. Antiplatelet medications are then considered. Choices include aspirin, clopidogrel, and prasugrel. NSTEMI treatment is rounded out with anticoagulants such as enoxaparin, bivalirudin, and fondaparinux. Based on risk stratification, definitive treatment may include medications-alone, angiography, percutaneous cardiac intervention or coronary artery bypass surgery. Clopidogrel acts by irreversibly inhibiting a platelet receptor that is needed for activation, thereby inhibiting platelet function. Thrombolytics (fibrinolytics), such as alteplase (A), reteplase and tenecteplase, are contraindicated in the treatment of NSTEMI, as they have shown worse outcomes with their use. Digoxin (C), a cardiac glycoside, is used in treating certain dysrhythmias and heart failure, not myocardial infarction. Beta-blockers such as metoprolol (D) carry a relative contraindication in patients with severe COPD, asthma, atrioventricular block, hypotension or bradycardia.

What medical condition is associated with Streptococcus bovis endocarditis?

Colorectal cancer.

Which of the following best describes the rhythm seen in the ECG above? Complete heart block First degree heart block Mobitz I (Wenckebach) Mobitz II

Complete heart block Third degree, or complete, heart block is characterized by absent conduction of all atrial impulses and complete electrical AV dissociation. The hallmark findings of third-degree AV heart block are regular PP intervals unrelated to regular R-R intervals with P waves that appear to march through the QRS-T complexes. There are two independent pacemakers: one in the S-A node and one either in the AV junction (narrow QRS complexes) or within the Purkinje fibers in the ventricles (wide QRS complexes). First degree heart block (B) is defined by normal AV conduction with a prolonged PR interval (>200 ms). There is a 1:1 relationship between P waves and QRS complexes. In Mobitz I (C), the PR interval progressively lengthens and the R-R interval progressively shortens until a beat is dropped. This cycle repeats itself, producing a pattern of "grouped beating." In Mobitz II (D), the PR interval is constant, but there are nonconducted P waves that lead to dropped beats. This block can deteriorate into third-degree heart block.

Which of the following conditions is associated with a pericardial knock on auscultation? Acute pericarditis Constrictive pericarditis Pericardial tamponade Restrictive cardiomyopathy

Constrictive pericarditis Constrictive pericarditis is associated with a pericardial knock. On cardiac auscultation, an early diastolic sound, the pericardial knock, may be heard at the apex 60 to 120 msec after the second heart sound (S2). The pericardial knock sounds like a ventricular gallop but occurs earlier than the S3 of heart failure, which it may mimic. The knock is due to accelerated right ventricular inflow in early diastole and early myocardial distention, followed by an abrupt slowing of further ventricular expansion. There is usually no pericardial friction rub. Constrictive pericarditis is pathologically distinct from acute pericarditis. Constrictive pericarditis is caused by the resultant inflammatory and reparative process from a pericardial injury. This leads to a fibrous thickening of the pericardium. Clinical presentation mimics heart failure and restrictive cardiomyopathy. Physical exam is associated with Kussmaul's sign (inspiratory neck vein distention) and pulsus paradoxus. Severe cases require surgical treatment with a pericardiectomy. Acute pericarditis (A) is associated with a pericardial friction rub. It is best heard with the diaphragm of the stethoscope at the lower left sternal border or apex when the patient is sitting and leaning forward or in the hands-and-knees position. The classic cardiac auscultation findings of pericardial tamponade (C) include distant heart or soft heart sounds. Restrictive cardiomyopathies (D) result from systemic disorders such as amyloidosis, sarcoidosis, hemochromotosis, scleroderma, carcinoid heart disease, and endomyocardial fibrosis. Findings on physical exam depend on the stage or severity of myocardial involvement. An S3 is almost always present, and an S4 is often heard.

While performing a routine physical examination, a provider notices a distinct decrease in the strength of the patient's radial pulse during inspiration. Which of the following conditions does this patient likely have? Constrictive pericarditis Diabetes mellitus Subclavian steal syndrome Tietze syndrome

Constructive pericarditis Pulsus paradoxus is an abnormally large decrease in systolic blood pressure and pulse wave amplitude during inspiration. The normal fall in pressure is less than 10 mm Hg. When the drop is more than 10 mm Hg, it is referred to as pulsus paradoxus. Pulsus paradoxus has nothing to do with pulse rate or heart rate. As a breath is taken in, the negative intrathoracic pressure within the chest increases venous return to the right side of the heart (increases right ventricular volume). Under normal circumstances, the right ventricle is able to expand into the pericardial space and has very little impact on the left ventricle. Pulsus paradoxus is a sign that is indicative of several conditions, including cardiac tamponade, constrictive pericarditis, chronic sleep apnea, croup, and obstructive lung disease. The paradox in pulsus paradoxus is that, on clinical examination, one can detect beats on cardiac auscultation during inspiration that cannot be palpated at the radial pulse. It results from an accentuated decrease of the blood pressure (due to reduced stroke volume), which leads to the radial pulse not being palpable and may be accompanied by an increase in the jugular venous pressure height (Kussmaul's sign). As is usual with inspiration, the heart rate is slightly increased, due to decreased left ventricular output. Diabetes mellitus (B), subclavian steal syndrome (C), and Tietze syndrome (D) are not associated with pulsus paradoxus.

Which virus is the most common cause of acute viral pericarditis?

Coxsackie virus.

What is the diagnostic murmur auscultated in aortic stenosis?

Crescendo-decrescendo systolic murmur.

What are some examples of newer oral anticoagulation agents?

Dabigatran, apixaban, rivaroxaban, edoxaban.

Orthostatic Hypotension

Decrease in systolic blood pressure ≥ 20 • Decrease in diastolic blood pressure ≥ 10 • Inadequate physiologic response to postural changes

You prescribe ramipril to a 65-year-old man with uncontrolled hypertension and severe renovascular disease. At a follow-up visit four weeks later his creatinine is noted to be 2.3. His creatinine before starting the ramipril was 1.0. By what mechanism did the ACE inhibitor cause this change? Activating the arachidonic acid pathway Decreasing glomerular blood flow Increasing angiotensin II activity Increasing serum kinin levels

Decreasing glomerular blood flow One of the actions of angiotensin II is to vasoconstrict efferent arterioles as they leave the glomerulus. This vasoconstriction stabilizes renal perfusion pressure and helps maintain normal creatinine levels. ACE Inhibitors block the conversion of angiotensin I to angiotensin II which causes a modest reduction in renal blood flow. This reduction may be severe in patients with bilateral renal artery stenosis, hypertensive nephrosclerosis, congestive heart failure, polycystic kidney disease or chronic kidney disease. In all of these cases renal perfusion is already compromised. By blocking angiotensin II with an ACE inhibitor, glomerular blood flow is decreased even further thus worsening the patient's creatinine. Activation of the arachidonic acid pathway (A) and increased serum kinin levels (D) are thought to be involved in the classic ACE inhibitor cough. Increased angiotensin II (C) activity would increase renal perfusion pressure and stabilize the creatinine, not worsen it.

A woman with known coronary artery disease presents to the ED with chest pain and ventricular tachycardia. Five minutes after admission she becomes unresponsive. Her rhythm strip is seen above. Which of the following is the most appropriate intervention? Beta-blockade Cardioversion Defibrillation Vasopressin

Defibrillation Ventricular fibrillation is the most common dysrhythmia in cardiac arrest patients. Instead of coordinated ventricular depolarization and contraction, ventricular fibrillation (VF) is characterized by rapid disorganized excitation potentials that amount to ineffective contraction. VF occurs with acute infarct or ischemia as well as old infarct or ischemia. Common triggers include electrocution, myocardial ischemia, and hypoxia. Electrocardiographically, VF appears as a chaotic, disorganized waveform that has no discernible typical morphology. Ventricular fibrillation is incompatible with life. Treatment is with immediate defibrillation. Vasopressin (D) was previously part of the ACLS protocol for pulseless ventricular tachycardia (pVT) and ventricular fibrillation (VF). However, the new 2015 AHA guidelines, have removed vasopressin all together. Beta-blockers (A) are not indicated for VF. Cardioversion (B) is only used in patients with organized electrical activity and who have a pulse.

What is beriberi?

Deficiency of thiamine (vitamin B1).

What are some triggers that can precipitate cardiac arrest in long QT syndrome?

Depending on the specific channel affected, various triggers can precipitate cardiac arrest including exercise, sudden loud noises or sleep.

A 68-year-old woman presents to the ED with chest pain. It occurs at rest, and has been getting worse over the past 2 hours. Her past medical history includes COPD, GERD, diabetes, urinary incontinence and Factor V Leiden. Her past surgical history is significant for carpal tunnel release four years ago and total hip arthroplasty 18 months ago. Blood pressure is 168/118 mm Hg, heart rate is 100 BPM and oxygen saturation is 95% on room air. An electrocardiogram reveals ST-elevation. An initial cardiac panel shows a positive troponin level. In this scenario, which of the following in this patient is a relative contraindication to fibrinolytic therapy? Diastolic blood pressure Factor V Leiden Gastroesophageal reflux disease Total hip arthroplasty

Diastolic blood pressure These symptoms are concordant with acute coronary syndrome, notably, acute ST-elevation myocardial infarction. While percutaneous coronary intervention is recommended, it is not always possible, due to facility abilities and services. In addition, fibrinolytic therapy is recommended, as ST elevation is indicative of complete coronary thrombotic obstruction. There are however absolute and relative contraindications to this type of therapy as the risk of internal hemorrhage could produce catastrophic events. Absolute contraindications include a history of any intracranial hemorrhage, cerebral vascular structural lesions or intracranial neoplasm. Others include ischemic stroke or head or facial trauma within the past 3 months and active bleeding or bleeding diathesis. Relative contraindications include systolic BP ≥ 180 or diastolic BP ≥ 110 mm Hg, as well as uncontrolled chronic hypertension, pregnancy and anticoagulant use. This patient's DBP is 118 mm Hg, above the recommended 100 mm Hg. Bleeding diatheses (tendency or predisposition), such as von Willebrand's disease and Factor V deficiency, are absolute contraindications to fibrinolytics. Factor V Leiden (B) is a pro-thrombotic coagulopathy, not a bleeding diathesis. As such, this is not a contraindication. Active peptic ulcer disease, not GERD (C), is a relative contraindication to fibrinolytic therapy. Major surgery, such as total hip arthroplasty (D), is a relative contraindication if it occurred within 3 weeks of acute coronary symptom onset.

Which value has greater influence on the mean arterial pressure, the diastolic or systolic blood pressure?

Diastolic blood pressure.

You are examining an afebrile 78-year-old woman in the emergency department. During cardiac examination, you auscultate a low intensity, low pitch extra heart sound which occurs in early diastole. You do not appreciate any murmurs. Her ECG appears normal. Which of the following is the most likely diagnosis? Bacterial endocarditis Dilated cardiomyopathy Right bundle branch block Tricuspid stenosis

Dilated cardiomyopathy The third heart sound (S3) is a low-frequency, brief vibration occurring in early diastole, at the end of the rapid diastolic filling period of the right or left ventricle. It is best appreciated at the apex in the left lateral position. It may be a normal variant in patients younger than age 40. After age 40, the presence of an S3 is usually abnormal, and correlates well with ventricular dysfunction, namely volume overload. However, any cause of ventricular dysfunction may be causative: dilated or ischemic cardiomyopathy, conduction abnormalities, left-to-right intracardiac shunts, ischemic heart disease, myocarditis or valvular regurgitation. High output states, such as anemia, thyrotoxicosis or pregnancy, also are causative. Although conduction abnormalities can cause ventricular dysfunction, bundle branch block (C) is unlikely in a patient with a normal ECG. Patients with bacterial endocarditis (A) are typically febrile. These patients usually have tricuspid or mitral valve vegitations. Tricuspid stenosis (D) is a rare complication of rheumatic fever. It is associated with a diastolic murmur.

Which two calcium channel blockers should be used most cautiously in conjunction with beta-blockers due to the increased risk of AV conduction depression?

Diltiazem and verapamil, the two nondihydropyridine calcium channel blockers, pose the greatest risk of depressing AV node conduction and sinus node automaticity when combined with beta-blockers.

A 67-year-old man with diabetes mellitus, chronic lower back pain and previously stable systolic heart failure now has increasing orthopnea and shortness of breath. Physical exam reveals pulmonary crackles, jugular venous distention and lower extremity edema. He is on carvedilol, lisinopril, furosemide, insulin and ibuprofen. Which of the following is appropriate for management of this patient? Add a calcium channel blocker Decrease the dosage of furosemide Discontinue the carvedilol Discontinue the ibuprofen

Discontinue the ibuprofen Non-steroidal anti-inflammatory drugs (e.g. ibuprofen) can worsen symptoms of heart failure. In fact, a number of medications that are in common clinical use are relatively or absolutely contraindicated in patients with heart failure. This is either because they can cause exacerbations of heart failure or because there is a higher risk of adverse events in such patients. Mechanisms by which some of these drugs can exacerbate heart failure relate to sodium retention, negative inotropic effects or direct cardiotoxicity. Non-steroidal anti-inflammatory drugs are associated with an increased risk of heart failure exacerbation, increased renal dysfunction and impairment of the response of angiotensin converting enzyme inhibitors and diuretics. Observational data has reported an association with non-steroidal anti-inflammatory drugs and increased mortality in patients with heart failure. It would not be correct to add a calcium channel blocker (A) as this class of medication has no direct role in the management of heart failure. Some studies have shown that their negative inotropic activity cause a greater clinical deterioration when compared to placebo, particularly with the first generation, non-dihydropyridines calcium channel blockers. It would also not be appropriate to decrease the dose of furosemide (B) as this patient is volume overloaded and needs diuresis to help alleviate his symptoms. It would not be appropriate to discontinue carvedilol (C) at this time. Although, generally, starting beta blocker therapy is not recommended in patients with decompensated heart failure, it is okay to continue beta blocker therapy if the patient has been previously stable on this medication.

An elderly woman presents with intermittent leg pain. She states it is a burning heaviness that is not necessarily associated with activity. Inspection reveals several dilated and tortuous veins about the lower legs. The skin is edematous and speckled with dark brown areas of capillary dilation but no specific pallor. Distal motor and sensory examinations are intact. Which of the following is the most appropriate initial tests in the evaluation of these symptoms? Angiography Coagulation panel Duplex ultrasonography Electrodiagnostics

Duplex ultrasonography Venous insufficiency, mainly due to incompetent or absent venous valves, can lead to retrograde blood flow in the superficial or deep venous systems. Ultimately, this leads to the syndrome of chronic venous insufficiency, which is marked by poor cosmesis, pain, lipodermatosclerosis, ulceration and life-threatening infections. The pain is usually described as burning, cramping or heaviness that occurs constantly in almost 20%, and episodically in almost 50% of sufferers. Chronic venous stasis or hypertension causes the characteristic skin changes of capillary proliferation, red or brown coloring, fat necrosis and fibrosis. These may be associated with edema, cellulitis, ulceration and cutaneous infarction. Although typical, these physical findings are only suggestive of the condition. Any suspicion is best evaluated initially with duplex ultrasonography. Angiography (A) is the test of choice for patients with suspected arterial insufficiency and peripheral arterial occlusion disorder. These conditions are more commonly associated with pain with activity and skin pallor, atrophy and shiny appearance. A coagulation panel (B) is usually performed as a baseline test before initiating anticoagulation in a patient with documented deep venous thrombosis. The above symptoms do not confirm, but merely suggest, venous insufficiency, which may be due to superficial or deep venous thrombosis. A diagnosis needs to be made before preparing for treatment. Electromyography and nerve conduction studies (D) are recommended for those with complaints and findings of distal neuropathy or myopathy, not venous insufficiency.

Heart Block: Third Degree

ECG will show P waves "march through" QRS complexes. • Treatment is pacemaker • Comments: P-P and R-R rate intervals will be regular without dropped beats

Heart Block: First Degree

ECG will show PR interval > 200 msecs with regular rhythm • Treatment is not necessary • Comments: most common conduction disturbance and usually represents an incidental finding on an ECG

Wolff-Parkinson-White (WPW) Syndrome

ECG will show short PR interval, delta wave, wide QRS • Most commonly caused by an accessory pathway (bundle of Kent) connects atria to ventricles, bypassing AV node • Definitive treatment is radiofrequency ablation

Left Bundle Branch Block

ECG will show: • Wide QRS >.12 sec • Broad, slurred R in V4 and V6 • Deep S in V1 and V2 • ST elevations in V1 - V3 • Comments: New LBBB + Chest Pain = MI until proven otherwise

Which of the following features can differentiate myocardial infarction from pericarditis? Chest pain ECG with reciprocal changes ECG with ST-segment elevations T wave flattening

ECG with reciprocal changes Reciprocal ST-segment depressions should never be seen in patients with pericarditis and an ECG with this finding should always be assumed to be from myocardial ischemia or infarction. There is no single best test for pericarditis but clinicians rely on the ECG as the best tool. Early in the disease, the ECG findings of acute pericarditis can mimic those of acute myocardial infarction (MI). The differentiation is critical as timely treatment of MI improves outcomes. Additionally, thrombolytic therapy should not be given to patients with pericarditis as it may precipitate hemorrhage into the pericardial space. Chest pain (A), ST-segment elevation (C) and T wave flattening (D) can be seen in both pericarditis and MI.

Which of the following will help to classify heart failure as being systolic or diastolic? Afterload Ejection fraction Heart rate Preload

Ejection fraction Ejection fraction is the percentage of blood that is ejected from the ventricle during systole. A normal ejection fraction is 55% or greater. Systolic dysfunction typically results from ischemic heart disease and myocardial cell death. It results in impaired contractility with an ejection fraction < 40%. Cardiac output is dependent on resistance (afterload) to emptying the ventricle. Diastolic dysfunction typically results from chronic hypertension and left ventricular hypertrophy. It results in impaired relaxation and ventricular filling with a normal ejection fraction. Output is dependent on ventricular filling (preload). Afterload (A) is the force needed to overcome both the volume of blood in the ventricle and the peripheral vascular resistance during ventricular contraction. The afterload is not specific for different types of heart failure. Heart rate (C) can also be variable in different types of heart failure. Preload (D) is the force or volume stretching the myocytes before contraction. It also can be thought of as the volume in the ventricle at the end of diastole just before the ventricle contracts. Output is dependent on preload in diastolic dysfunction, but it poorly differentiates diastolic from systolic heart failure.

What is the name of the ECG finding classically associated with large pericardial effusions?

Electrical alternans.

What is the treatment of hemodynamically unstable ventricular tachycardia?

Electrical cardioversion.

Which of the following is the most common cause of tricuspid regurgitation? Bacterial endocarditis Elevated right heart pressure Myxomatous degeneration Rheumatic heart disease

Elevated right heart pressure Tricuspid regurgitation refers to incompetence of the tricuspid valve leaflets such that some blood flows backwards from the right ventricle into the right atrium during systole, producing a blowing holosystolic murmur best heard at the left sternal border. Like all right heart murmurs, the murmur of tricuspid regurgitation is enhanced by inspiration. Tricuspid regurgitation is most commonly functional, caused by dilation of the right atrium or ventricle and subsequent stretching of the annulus of the valve. Pathologic tricuspid regurgitation occurs most often due to elevated right heart pressure, typically due to pulmonary problems such as COPD or pulmonary fibrosis, or volume overload. Tricuspid regurgitation can also result from seeding of the tricuspid valve with bacteria as seen in endocarditis, creating vegetations that render the valve leaflets incompetent. This is usually due to intravenous drug use, which introduces aggressive bacterial pathogens like S. aureus into circulation and the right heart. Bacterial endocarditis (A), especially from intravenous drug use, can cause tricuspid valve vegetations and incompetence. Still, primary tricuspid valve disease is less common than functional tricuspid valve disease. Myxomatous degeneration (C) is most commonly the cause of mitral valve prolapse with or without associated mitral valve regurgitation but may also be associated with the much more rare tricuspid valve prolapse. Rheumatic heart disease (D) is the most common cause of primary valvular heart disease in the non-industrialized world and usually causes mitral stenosis in the chronic setting. Tricuspid valve involvement is uncommon.

A 21-year-old woman presents with shortness of breath, rash and nausea after an insect bite. Her vitals are T 97.7°F, HR 128, BP 85/56, RR 28, oxygen saturation 93%. Exam reveals diffuse hives and posterior pharyngeal swelling. Which of the following should be immediately administered? Epinephrine 1:10,000, 0.3 mL IM Epinephrine 1:10,000, 10 mL IV Epinephrine 1:1000, 0.3 mL IM Epinephrine 1:1000, 0.3 mL IV

Epinephrine 1:1000, 0.3 mL IM This patient presents with anaphylactic shock from an insect bite and requires immediate administration of epinephrine. Epinephrine is potentially life-saving in severe anaphylactic reactions. During an anaphylactic reaction, mast cells degranulate leading to release of histamines and other immune mediators. These mediators lead to the hallmark symptoms of allergic reactions including hives, nausea and vomiting, airway edema, bronchoconstriction and hypotension. Epinephrine acts immediately on adrenergic receptors to reverse these symptoms. Inhaled beta agonists can also be given to rapidly reverse bronchoconstriction. Many of the other treatments in anaphylaxis have a delayed onset of action. Diphenhydramine (H1 receptor blocker) acts by blocking the effect of histamine on H1 receptors mainly in the skin. Ranitidine, famotidine and other H2 blockers can mitigate some of the gastrointestinal symptoms caused by histamine as well as some of the cutaneous manifestations. Steroids mainly act by stabilizing mast cells from further degranulation but this effect is usually delayed 4-6 hours after administration. Steroids also increase the expression of beta-receptors in the lung increasing the efficacy of inhaled beta-agonists. Epinephrine should be given as 300 - 500 mcg IM in the anterolateral thigh. This dose equates to 0.3 - 0.5 mL of the 1:1000 preparation. Giving epinephrine 1:1000, 0.3 mL IV (D) is a massive overdose and can cause dysrhythmias and cardiac ischemia. Cardiac epinephrine (A & B) (1:10,000 concentration) comes in vials of 10 mL for a total of 1000 mcg of epinephrine. This preparation of epinephrine is reserved for use in patients with cardiac arrest (ventricular fibrillation, pulseless electrical activity or asystole) and should not be given to patients with a blood pressure. 0.3 mL of cardiac epinephrine IM is a 10-fold underdose as this amount only contains 30 mcg of epinephrine.

Which drugs have been implicated as a possible cause of mitral regurgitation?

Ergotamine, bromocriptine, cabergoline, and anorectic drugs.

Which of the following drugs is most likely to be associated with the development of atrial tachydysrhythmias? Ethanol Gamma hydroxybutyrate (GHB) Lorazepam Phenobarbital

Ethanol Ethanol abuse is associated with the development of atrial dysrhythmias, specifically, atrial fibrillation. Alcohol ingestion (acute or chronic) has multiple effects on the cardiovascular system. It can exacerbate coronary artery disease, lead to cardiomyopathy and produce dysrhythmias. Left ventricular dysfunction is common in patients with moderate alcohol consumption. Additionally, these patients may have diastolic dysfunction. Supraventricular and ventricular dysrhythmias are common. The so called "holiday heart" that occurs with heavy drinking can present as atrial fibrillation or, in unusual cases, ventricular tachycardia. Additionally, electrolyte deficiencies (hypokalemia and hypomagnesemia) predispose to dysrhythmias. GHB (B) has been associated with bradycardia and hypotension. Lorazepam (C) and phenobarbital (D) are generally not associated with cardiac dysrhythmias.

How often should patients be screened who are below the threshold for lipid-lowering therapy?

Every five years.

Acute Decompensated Heart Failure

Exertional dyspnea, orthopnea, paroxysmal nocturnal dyspnea, pitting edema S3 ↑ BNP CXR: cardiomegaly, cephalization, Kerley B lines, effusions Most useful study: echo Treatment: BiPAP: ↑ oxygenation, ↓ work of breathing, ↓ preload/afterload Nitroglycerin: ↓ preload/afterload Morphine: ↓ myocardial O2 consumption, ↓ preload Furosemide: diuresis Hypotension without signs of shock: dobutamine (may worsen hypotension) Severe hypotension with signs of shock: norepinephrine (↑ systemic vascular resistance, ↑ HR, ↑ BP, ↑ myocardial O2 demand)

Premature Atrial Contractions

Extra beats that originate outside sinus node • Originate from ectopic atrial pacemaker • Different morphology from sinus P waves • Appears interspersed throughout an underlying rhythm

Which of the following is a minor Jones criteria for the diagnosis of acute rheumatic fever? Carditis Fever Polyarthritis Sydenham chorea

Fever Fever is one of four minor Jones criteria for the diagnosis of acute rheumatic fever (ARF). ARF results from molecular cross-reactivity between streptococcus and host cell proteins resulting in antibody formation. This is a type II hypersensitivity inflammatory reaction. Diagnosis is based on the Jones criteria, which contains major and minor criteria. A patient is determined to have ARF if they have evidence of a prior group A streptococcus infection along with either two major criteria or one major and two minor criteria. Aside from fever, the other minor criteria are arthralgia, elevated ESR/CRP, and prolonged PR interval. Patients should be investigated for rheumatic heart disease. Treatment usually starts with treating residual group A streptococcal infection and treatment of pain and inflammation with non-steroidal anti-inflammatory agents, salicylates and steroids. Carditis (A), polyarthritis (C), Sydenham chorea (D), erythema marginatum, and subcutaneous nodules are all major Jones criteria.

Constrictive Pericarditis

Fibrotic changes leading to reduced diastolic filling No wall enlargement Pericardial knock on cardiac auscultation Kussmaul's sign and pulsus paradoxus Pericardiectomy

Name two medications used to treat orthostatic hypotension?

Fludrocortisone and pyridostigmine.

A 72-year-old man presents for evaluation of palpitations. He has a regular, wide complex tachycardia at a rate of 140 bpm. Which of the following supports a diagnosis of ventricular tachycardia? Discordance of the QRS axis in the precordial leads Fusion beats Leftward axis ST elevation greater than 5 mm

Fusion beats Ventricular tachycardia (VT) originates from a location within the ventricle typically outside of the normal conduction system. Increased automaticity or a reentry circuit may cause it. The majority of patients with VT have underlying cardiac disease. The most common form of VT is monomorphic where the QRS complexes appear the same morphologically. Since the electrical impulse originates outside of the conduction system, the QRS complex is wide. It is sometimes challenging to differentiate between VT and other wide complex tachycardias (eg, supraventricular tachycardia with aberrancy). VT is typically regular although there may be a small amount of irregularity helping to distinguish between it and supraventricular tachycardia. Fusion beats occur when impulses from two different locations (one within the ventricle and one in a supraventricular location) activate the ventricle. The result is a QRS complex with morphology resembling a hybrid of a sinus beat and intraventricular beat. These are diagnostic of VT because they represent AV dissociation. Capture beats occur when a sinus beat is normally conducted and a single beat with the sinus QRS morphology occurs within a wide complex tachycardia. Discordance of the QRS axis in the precordial leads (A) is NOT suggestive of VT. Typically across the precordium in VT the QRS complex demonstrates concordance or its polarity. A leftward axis (C) is not suggestive of VT and a more characteristic axis deviation is right superior. ST elevation greater than 5 mm (D) is not part any diagnostic criteria for the identification of VT.

A 64-year-old man with a history of hypertension presents to the Emergency Department requesting medication refills. He states that he has not taken his medications for the last 2 weeks. His blood pressure is 190/100. He has no complaints at this time. He has prescription bottles for atenolol and hydrochlorothiazide. What management is indicated? Change his medications to a calcium-channel blocker Give the patient a prescription for his medications and refer to his primary doctor in 48 hours Start intravenous labetalol and admit to the floor Start intravenous labetalol and admit to the intensive care unit

Give the patient a prescription for his medications and refer to his primary doctor in 48 hours This patient presents with asymptomatic hypertension in the setting of medical non-compliance and should be restarted on his medications and scheduled for follow up with a primary care provider. Accelerated hypertension is defined as markedly elevated blood pressure in the absence of symptoms. This is in contrast with hypertensive emergency where the patient has symptoms or evidence of end organ system dysfunction or both as a result of elevated blood pressure. Accelerated hypertension has a poor long-term prognosis if not controlled but does not pose an immediate threat. As such, it should not be aggressively treated with parenteral medications. Rapid lowering of blood pressure in patients with chronic elevated blood pressure can cause organ hypoperfusion, particularly brain hypoperfusion, and lead to serious sequelae. These patients should be restarted on their medications (if appropriate) and sent for follow up with a primary care physician to monitor and treat the elevated blood pressure. There is no reason to change the patient's medications (A) since he has not been taking them. Starting an intravenous medication (C and D) is required in the treatment of hypertensive emergency but may be dangerous in patients with asymptomatic elevated blood pressure.

What is the goal heart rate during an treadmill exercise stress test?

Goal= 85% of maximum predicted heart rate [220 - patient age].

Which gram-negative organisms which are difficult to culture can cause endocarditis?

HACEK group (Haemophilus, Actinobacillus, Cardiobacterium, Eikenella, and Kingella).

Which of the following is the most common physical finding in patients with infective endocarditis? Heart murmur Janeway lesion Osler nodes Splinter hemorrhages

Heart murmur Infective endocarditis occurs when pathogens introduced to the systemic circulation invade the endocardial surface of the heart, including the heart valves. Staphylococcal and Streptococcal bacteria are the most common culprits. The clinical signs and symptoms of endocarditis are varied. Symptoms include fever, chills, dyspnea, weakness, nausea, vomiting, cough, and chest pain. Regarding physical findings, fever is the most common abnormality, seen in 90%. The majority of patients with endocarditis will have a heart murmur (85%). Approximately half of patients with endocarditis develop embolic phenomena, where septic microemboli break off and lodge in downstream tissues. Characteristic skin findings are somewhat less common. Osler nodes, tender lesions on the pads of the fingers, are seen in 10-23% of patients with endocarditis. Janeway lesions, nontender erythematous lesions on the extremities, are seen in 10%, and splinter hemorrhages, linear lines in under the nails, are seen in 15% of patients. The treatment of infective endocarditis involves antibiotic therapy targeted to the suspected pathogen, and stabilization of any hemodynamic instability. Janeway lesions (B), Osler nodes (C), and splinter hemorrhages (D) are relatively uncommon physical findings of infective endocarditis.

A 28-year-old woman with no past medical history presents to the emergency department with acute dyspnea. Physical exam reveals tachycardia, warm extremities, wide-pulse pressure, bounding pulses, a systolic flow murmur, exophthalmos and a neck mass. Which of the following is the most likely diagnosis? Aortic regurgitation High output heart failure Low output heart failure Methamphetamine intoxication

High output heart failure This patient most likely has high-output heart failure secondary to thyrotoxicosis. High output heart failure occurs when cardiac output is elevated in patients with reduced systemic vascular resistance. Examples include thyrotoxicosis, anemia, pregnancy, beriberi and Paget's disease. Patients with high output heart failure usually have normal pump function, but it is not adequate to meet the high metabolic demands. In high output heart failure the heart rate is typically elevated, the pulse is usually bounding and the pulse pressure wide. Pistol-shot sounds may be auscultated over the femoral arteries, which is referred to as Traube's sign. Subungual capillary pulsations, often referred to as Quincke's pulse, may also be present. Although these findings may be seen in other cardiac conditions, such as aortic regurgitation or patent ductus arteriosus, in the absence of those conditions, these signs are highly suggestive of elevated left ventricular stroke volume due to a hyperdynamic state. Patients with chronic high output also may develop signs and symptoms classically associated with the more common low-output heart failure; specifically, they may develop pulmonary or systemic venous congestion or both, while maintaining the above normal cardiac output. Low output heart failure (C) is often secondary to ischemic heart disease, hypertension, dilated cardiomyopathy, valvular and pericardial disease or arrhythmia. It can cause dyspnea but is not associated with symptoms of a hyperdynamic state. Aortic regurgitation (A) is classically associated with bounding pulses, a wide pulse pressure and subungual capillary pulsations; however, aortic regurgitation is less likely in a young woman with no past cardiac history. Additionally, this woman has exophthalmos and a goiter on exam, which support the diagnosis of thyrotoxicosis. Methamphetamine intoxication (D) usually presents with agitation, tachycardia, and psychosis; however, it is not associated with a hyperdynamic state, exophthalmos or a goiter.

A 35-year-old woman with alcohol abuse presents to clinic complaining of "skipped heart beats." Her examination is normal. An in-office ECG is also normal. Laboratory testing is nonspecific. Which of the following is the next most appropriate test? Echocardiogram Electromyography Holter monitor Sestamibi scan

Holter monitor Premature atrial contractions (PACs) are extra beats that originate outside the sinus node from ectopic atrial pacemakers. They appear interspersed throughout an underlying rhythm, usually sinus. Common symptoms include palpitations, described by many as a skipped beat or a missing beat. Others describe it as a stopping or flipping of the heartbeat. These symptoms mimic those of premature ventricular contractions. Causes include psychological stress, hypertension, valvulopathy, ischemic cardiac disease, stimulants (caffeine, tobacco, alcohol), digitalis toxicity, electrolyte abnormalities, and idiopathic. Most cases are asymptomatic and occur in otherwise healthy hearts. Many times one can only find them with continuous 24 hour monitoring via a Holter monitor. PACs are benign, however, a small amount may deteriorate into atrial flutter or fibrillation as well as supraventricular tachycardia. Although echocardiography (A) is usually part of the evaluation of PACs, one must first diagnose them by finding them on electrocardiography. Electromyography (B) is used in the evaluation of nerve and skeletal muscle disorders, not cardiac disorders. Sestamibi scanning (D) is a nuclear medicine test used in the evaluation of myocardial ischemia and infarction. It is not the next most appropriate test when the clinician does not have a working diagnosis of "skipped beats."

What is the earliest change seen on electrocardiogram in an acute ST elevation myocardial infarction? Hyperacute T waves J point elevation PR depression ST segment elevation

Hyperacute T waves Electrocardiogram abnormalities during an acute myocardial infarction follow a typical progression. The earliest ECG finding is the hyperacute T wave. They are usually broad based and slightly asymmetrical. Depending on the timing of presentation, this finding may not be seen. As infarction progresses, ST segment elevation becomes apparent in leads specific to the coronary artery involved. Reciprocal ST depression will be seen in electrically opposite leads (e.g. an inferior myocardial infarction will present with ST elevations in leads II, III and aVF with reciprocal ST depression in lead aVL and possibly lead I). Q waves are commonly seen after the development of ST segment elevation and may persist as a marker of previous infarction. The J point (B) is found at the end of the QRS complex and the beginning of the ST segment. A positive deflection of the J point is termed an Osborn wave and can be seen in hypothermia. Notching of the J point can be seen in benign early repolarization. PR depression (C) is not a typical component of a STEMI but can be seen in pericarditis. ST segment elevation (D) is the hallmark of acute myocardial infarction but is not the first change seen on an ECG.

Hypertensive Emergency

Hypertension with acute end-organ system injury Encephalopathy, cardiac ischemia, renal ischemia Objectives: reduce MAP 25% in first hour, normalize BP over the next 8 to 24 hours Reduction of MAP > 25% may cause end-organ ischemia IV antihypertensives (labetalol or nicardipine)

A man with dyspnea-on-exertion presents for cardiac evaluation. Physical exam is significant for a left sternal border systolic murmur which is louder during a Valsalva maneuver. An S4 is also appreciated. The ECG shows large QRS complexes. An echocardiogram reveals a decrease in left ventricular chamber volume and normal left atrial measurements. Which of the following is the most likely diagnosis? Aortic stenosis Dilated cardiomyopathy Hypertrophic cardiomyopathy Mitral stenosis

Hypertrophic cardiomyopathy Hypertrophic cardiomyopathy (HCM) results from left or right ventricular hypertrophy or both. This condition can be primarily caused by autosomal dominant genetic mutations of the cardiac sarcomere genes and myocardial fiber hypertrophy. Secondary causes include aortic stenosis, mitral valve abnormalities, coronary heart disease and chronic systemic hypertension. Most patients are asymptomatic upon presentation, however, the common clinical manifestations are dyspnea, angina and dysrhythmia. Decreased chamber volume and increased ventricular wall thickness are the key echocardiographic findings in HCM. Furthermore, the ECG typically shows left ventricular hypertrophy, T-wave inversion and large QRS complexes. A harsh, left sternal border, systolic, crescendo-decrescendo murmur which is worse with Valsalva maneuver is quite typical of HCM. Aortic stenosis (A) is associated with a right, not left, sternal border pansystolic murmur that is decreased, not increased, during a Valsalva maneuver. S3, normal or thin ventricular wall and enlarged ventricular chamber volume are more common with dilated cardiomyopathy (B). S4, thick ventricular wall and decreased ventricular chamber volume are more common with hypertrophic cardiomyopathy. Mitral stenosis (D) produces a diastolic, not systolic, murmur, and is associated with increased left atrial size and pressure.

What are the two major side effects of procainamide?

Hypotension and prolonged QT interval.

A 47-year-old man presents to the emergency department due to acute, sharp chest pain that is relieved by leaning forward. A pericardial friction rub is auscultated on cardiac examination and an ECG shows diffuse ST elevations. Echocardiography reveals a small pericardial effusion. Which of the following is the most appropriate initial treatment? Atenolol Ibuprofen Nitroglycerin Prednisone

Ibuprofen Pericarditis is defined as an inflammation of the pericardium and can be caused from an acute myocardial infarction, viral etiologies, medications, systemic disease and trauma to the chest cavity. A pericardial friction rub is often heard on auscultation and the patient presents with retrosternal chest pain that is relieved by leaning forward. Classic electrocardiographic changes include widespread concave upward ST-segment elevation without reciprocal T-wave inversions or Q waves. First-line treatment includes nonsteroidal anti-inflammatory drugs and colchicine. Beta-blockers such as atenolol (A) are used for cardiac arrhythmias such as atrial fibrillation. They are also used in the post myocardial infarction regimen as they help in the remodeling of the cardiac muscle. Nitroglycerin (C) is used for acute chest pain in the setting of a myocardial infarction as it reduces the afterload. Corticosteroids such as prednisone (D) are considered a second-line treatment for pericarditis when NSAID's and colchicine fail to relieve symptoms in severe or refractory cases that are often marked by systemic disease.

Which of the following medications can be used in the treatment of hemodynamically stable atrial fibrillation with concomitant Wolff-Parkinson-White syndrome? Adenosine Diltiazem Ibutilide Metoprolol

Ibutilide Ibutilide can be used for rhythm control in hemodynamically stable atrial fibrillation with concomitant Wolff-Parkinson-White syndrome, which is also referred to as preexcited atrial fibrillation. Atrial fibrillation can occur in up to one third of patients with Wolff-Parkinson-White syndrome. Tachydysrhythmias can be facilitated by direct conduction from the atria to the ventricles via the accessory pathway, bypassing the AV node, as seen with atrial fibrillation or atrial flutter in conjunction with Wolff-Parkinson-White. Treatment with AV nodal blocking drugs such as adenosine, calcium-channel blockers, beta-blockers and digoxin may increase conduction via the accessory pathway with a resultant increase in ventricular rate and possible degeneration into ventricular tachycardia or ventricular fibrillation. The goals of acute drug therapy for preexcited atrial fibrillation are prompt control of the ventricular response and, ideally, termination of atrial fibrillation. If the patient is unstable because of a rapid ventricular response, electrical cardioversion should be performed. For more stable patients, trials of intravenous medications can be performed cautiously. Treatment of preexcited atrial fibrillation requires a parenteral drug that lengthens antegrade refractoriness and slows conduction in both the AV node and the His-Purkinje system as well as the accessory pathway. Ibutilide is a class III antiarrhythmic drug that prolongs the refractoriness of the AV node, His-Purkinje system, and accessory pathway, and is useful for termination of atrial fibrillation and atrial flutter. If ibutilide is not available, or if the concern for drug-induced QT prolongation and polymorphic ventricular tachycardia is high, procainamide in another option. Adenosine (A), Diltiazem (B), and Metoprolol (D) are all AV nodal blocking agents and should be avoided in patients with preexcited atrial fibrillation. Blocking the AV node can promote and enhance conduction down the accessory pathway, which can deteriorate into an unstable rhythm such as ventricular tachycardia or ventricular fibrillation.

Acute Pericarditis

Idiopathic > viral (Coxsackie) • Pleuritic chest pain radiating to the back • Pain ↓ with leaning forward • Pericardial friction rub • ECG: diffuse STE, PR depression • NSAIDs

Anaphylactic Reaction

IgE-mast cell mediated • Airway management • Antihistamines, dexamethasone, IM epinephrine, IVF • Glucagon for refractory hypotension in patient with known HTN

Other than magnesium sulfate and cardioversion/defibrillation, what is another treatment for torsades de pointes?

Increase the heart rate to shorten ventricular repolarization, also known as overdrive pacing.

A 42-year-old man presents to the Emergency Department with fever, chills, cough, and hemoptysis. He has a history of intravenous opioid use. Vital signs include BP 110/65 mm Hg, HR 120 beats per minute, RR 20 breaths per minute, and T 103.4°F. Chest X-ray is shown above. Which of the following is the most likely diagnosis? Diffuse alveolar hemorrhage Infectious endocarditis Miliary tuberculosis Wegener's granulomatosis

Infectious endocarditis This patient is presenting with signs and symptoms of infectious endocarditis. Risk factors for infectious endocarditis include rheumatic heart disease, congenital or acquired valvular disease, and intravenous drug use. Infectious endocarditis is classified as acute or subacute depending upon the time course and presentation. Acute endocarditis more often affects normal valves in younger patients. Septic emboli (as shown in the above chest X-ray) and significant illness are common on presentation. Subacute endocarditis has a predilection for abnormal valves and more frequently occurs in older patients. These patients are typically less ill on presentation with intermittent fevers and constitutional symptoms. Left-sided endocarditis involves either the aortic or mitral valve. It is more common than right-sided endocarditis. Organisms often implicated in left-sided endocarditis include Streptococcus viridans, Staphylococcus aureus, and those in the Enterococcus family. Complications include systemic infarcts from septic emboli. Right-sided endocarditis involves either the pulmonic or tricuspid valve. It is classically seen in intravenous drug users. Organisms implicated in right-sided endocarditis include Staphylococcus aureus, Streptococcus pneumoniae, and gram negative bacteria. Presenting symptoms often include fever, cough, hemoptysis, chest pain, and dyspnea. Right-sided endocarditis is frequently misdiagnosed initially as pneumonia. Dermatologic and ocular manifestations of endocarditis are important indicators of the diagnosis. Roth spots are retinal hemorrhages with central clearing seen on funduscopic examination. Osler nodes are painful nodules on fingers and toes. Janeway lesions are painless erythematous plaques on the palms and soles. Splinter hemorrhages occur beneath the nails due to septic emboli. Diagnosis is made by having either both major criteria, 1 major and 3 minor criteria, or 5 minor criteria. Major criteria include 2 positive blood cultures with at least 3 sets sent one hour apart of organisms common to infectious endocarditis or abnormal echocardiography with either visible vegetation, new valvular regurgitation, prosthetic valve dehiscence, or myocardial abscess. Echocardiography is the hallmark of imaging for endocarditis and is preferably done via the transesophageal route. Minor criteria include predisposing risk factors or IV drug use, fever, vascular events such as septic emboli and Janeway lesions, immunologic events such as Osler nodes or Roth spots, echocardiographic findings consistent with endocarditis not meeting major criteria, and positive blood cultures not meeting major criteria. Management includes antibiotics for the suspected organism based on the clinical situation. In an IV drug user, coverage should include methicillin-resistant Staphylococcus aureus and Pseudomonas aeruginosa in addition to the typically implicated organisms. The most appropriate antibiotics for this patient would include cefepime and vancomycin. Diffuse alveolar hemorrhage (A) will often present with cough and hemoptysis; however, fever is uncommon. Chest X-ray will typically show bilateral infiltrates, primarily central. The classic finding in diffuse alveolar hemorrhage is return of increasing amounts of blood on bronchoalveolar lavage. Miliary tuberculosis (C) will often present with a diffuse reticulonodular pattern on chest X-ray as well as generalized malaise and fever, but rarely with hemoptysis. Also, intravenous drug use is a much more significant risk factor for infectious endocarditis. Wegener's granulomatosis (D) will often present with chronic low-grade fevers, night sweats, fatigue, and weight loss. Patients will often have chronic sinusitis, but not present until they have severe hemoptysis, vasculitis, or oral lesions. Chest X-ray will often show bilateral central infiltrates; however, the patient will not present with high fever and intravenous drug use is not a risk factor for this disease.

A previously healthy 27-year-old woman presents to your clinic with concerns about screening for cholesterol. She has never smoked, is physically active, and is in the normal range for body mass index and blood pressure. She says that her father started taking medication for hyperlipidemia at age 66 and she wants to know when she should start being screened. Which of the following is the most appropriate next step in management? Initiate lipid screening at age 35 Initiate lipid screening at age 45 Initiate preventative therapy with aspirin Initiate preventative therapy with lovastatin

Initiate lipid screening at age 45 Screening for lipid abnormalities is a part of determining overall cardiovascular risk. Lipid-lowering therapy is one way to improve cardiovascular outcomes and patients at risk should be screened to evaluate the need for treatment with aspirin or statins. The need for and timing of screening is determined by assessing patient risk factors including age, sex, and other risk factors for cardiovascular disease such as smoking, hypertension and family history of premature coronary heart disease. Patients are at higher risk if they have more than one risk factor or a single risk factor that is severe. Examples of severe single risk factors include a heavy smoking history or multiple first-degree family members with coronary heart disease at a young age. Women determined to be at low risk should start screening for lipid abnormalities at age 45. Men at low risk should start screening at age 35. Women determined to be at a higher cardiovascular risk by the presence of more than one risk factor or a severe single risk factor should start lipid screening at age 35 (A). Men at high risk should start screening at age 25. Therapy with aspirin (C) and statins (D) is recommended with a diagnosis of hyperlipidemia and not as a preventive measure.

A 28-year-old man presents to the emergency department with a one cm laceration to the left forearm. The patient is in good health and has no other complaints. At triage the patient's blood pressure was noted to be 155/94 mmHg; the remainder of his vital signs are normal. Following repair of the laceration, the patient's blood pressure is rechecked and is unchanged. What is the best approach to this patient's elevated blood pressure? Administer labetalol or nifedipine with observation until BP is <140 systolic Evaluate for end organ damage with chest X-ray, ECG, electrolytes, BUN/creatinine and urinalysis Instruct the patient to follow-up with his private physician within two months for recheck No further management required

Instruct the patient to follow-up with his private physician within two months for recheck All patients seen for emergency care, regardless of the complaint should have their blood pressure recorded. A vast majority of the hypertension encountered in the ED is either transient or mild. The most common cause of transient hypertension is pain and anxiety. In these patients, end-organ ischemia does not occur, and attention is focused on treatment of the primary process. If the pressure is elevated, it should be repeated prior to discharge. The patient is advised of this potential problem and appropriate follow up arranged. In adults, systolic blood pressure less than 120 mm Hg and diastolic pressure of less than 80 mm Hg are considered normal. If the diastolic pressure is between 80 to 104 mm Hg, the patient should have follow-up within two months. For diastolic blood pressure greater than or equal to 115 mm Hg, the patient should be evaluated immediately. The patient does not require diagnostic workup (B) or therapeutic intervention (A). The patient should be informed of his or her blood pressure and arrangements should be made for outpatient follow up (D).

Atrial Fibrillation

Irregularly irregular No P waves Narrow QRS unless conduction block or accessory pathway Variable ventricular response rate

Which of the following best describes the underlying pathophysiology of ventricular tachycardia? Calcified mitral valve leaflets Electrical blockade in the atrioventricular node Irritable ventricular foci Reentrant pathway exists between atria and ventricles

Irritable ventricular foci Ventricular tachycardia (VT) is usually initiated by irritable ventricular automaticity foci. These foci may become irritable by several factors, three of which are hypokalemia, hypoxia and ischemia. Hypoxia may result from airway obstruction, lack of air (as in suffocation or drowning) and pulmonary compromise (as in pulmonary embolus or pneumothorax). Myocardial ischemia or infarction occurs in disease states such as chronic ischemic heart and coronary disease, or in emergent states, as in hypovolemic shock or cardiogenic shock. The more common causes of irritation are coronary insufficiency (vasospasm as with cocaine, atherosclerosis or thrombosis and embolus) and myocardial infarction. Once a ventricular foci is irritated enough, it will take over as the dominant pacemaker, suppressing the sinoatrial node, and resulting in a tachycardia usually between 150-250 bpm. Electrical blockade thru the atrioventricular node (B) is the basis of AV block, not ventricular tachycardia. Mitral stenosis is caused by rheumatic fever and leaflet calcification (A). A diseased mitral valve is associated with atrial fibrillation. A reentrant pathway between atria and ventricles (D) explains the basis of supraventricular tachycardia (SVT).

What is the most common cause of heart failure in the United States?

Ischemic heart disease

What effect will the administration of atropine have on third degree heart block?

It alters conduction ratios without changing the appearance of the ventricular escape rhythm.

Does the volume of a pericardial friction rub increase or decrease with inspiration?

It increases during inspiration.

What is a Still's murmur?

It is a short, vibrating, physiologic murmur which can be heard over the mid precordium in children in the absence of any other abnormality. This is a common and benign condition.

18 months ago, an elderly patient received a mitral valve replacement. Unfortunately, for the past year, he has been fighting subacute bacterial endocarditis. He has been admitted to the hospital 3 times in the past 6 months. Of all things, he is most concerned with unsightly changes of his palms. During inspection, you appreciate that both palms have several nontender macules of red to brown to black coloration. His daughter is getting married in 3 weeks, and he doesn't want people to see these "weird rashes" on his hands. Which of the following correctly names these findings? Janeway lesions Raynaud's phenomenon Roth spots Splinter hemorrhages

Janeway lesions Bacterial endocarditis usually presents with persistent fever, bacteremia and constitutional symptoms. There are also several physical examination findings which clue the clinician into considering this diagnosis. A new onset murmur, a change in a previous murmur or a new thrill are common presenting signs. Conjunctival and palate petechiae are also common. Tender splenomegaly or tender joints may be present. Neurologic symptoms may be the result of septic emboli emanating from an acutely infected cardiac valve. Septic emboli can also present with cutaneous findings, namely Janeway lesions, however, these are usually present in subacute rather than acute endocarditis. They are described as multicolored hemorrhagic macules which appear on the palms and soles. Treatment of prosthetic valve endocarditis of >1 year's duration is vancomycin plus gentamicin plus ceftriaxone, adjusted, of course, to culture results. Raynaud's phenomenon (B) commonly occurs in rheumatologic disease, like systemic sclerosis. It represents a cold-induced distal digital vasoconstriction, which causes finger, not palmar, pallor, cyanosis or hyperemia and pain. Roth spots (C), also a common finding in bacterial endocarditis, represent retinal hemorrhages surrounding pale centers. They do not occur on the palms. Other cutaneous findings of bacterial endocarditis include proximal nail bed, not palmar, splinter hemorrhages (D).

When does the vessel-expansion, typically seen in the jugular vein as a double-pulsation, occur in the cardiac cycle?

Just after S1 and during S2.

A 58-year-old man with chronic hypertension presents to the ED with acute, 10/10 tearing substernal pain that radiates to the back. All you can gather from him is that he also has some type of "collagen disorder" and diabetes. A chest radiograph reveals a widened mediastinum. As you prepare for a transesophageal echocardiogram, you would most likely start which of the following medications as a first-line agent? Clonidine Labetalol Lisinopril Nitroprusside

Labetalol A classic aortic dissection involves an intimal tear and hemorrhagic extravasation into the intima-media space. Aortic dissections can be defined as proximal, affecting the ascending aorta, or distal, involving the descending aorta distal to the subclavian take-off. Several risk factors of this potentially fatal condition exist, and include hypertension, congenital aortic valve disorder (coarctation, root-dilatation or bicuspid valve), trauma, cardiac surgery, aortitis and connective tissue disorder. Both types are characterized by severe, "ripping or tearing" pain that is maximal at onset (as compared to the crescendo pain of an acute coronary syndrome) and located in the chest, back or abdomen. A new aortic insufficiency murmur is more common in proximal dissection, while a history of hypertension is more common in the distal type. Evaluation includes chest radiograph, chest CT and transesophageal echocardiogram. First-line therapy is intravenous beta-blockers, such as labetalol or esmolol, followed by vasodilators like nitroprusside. Emergency surgery is very likely, especially for proximal dissections. Alpha-2-agonists, like clonidine (A), and angiotensin-converting enzyme inhibitors, like lisinopril (C), are not the recommended treatment of aortic dissection. The goal in treating aortic dissection is to first use beta-blockers, and then use vasodilators, like nitroprusside (D). Administering beta-blockers first helps to protect against reflex tachycardia and positive inotropy that sometimes occurs with agents that cause profound vasodilation.

A 14-day old boy is brought to clinic for a well-child check. The boy was born full-term via normal spontaneous vaginal delivery to a G2, P2 mother who had limited prenatal care. There were no complications at delivery. The boy has not yet regained birth weight and has been breastfeeding poorly. On physical examination, you note bluish discoloration of the lips and oral mucosa, clear breath sounds and a harsh left upper sternal border murmur. Which of the following chest X-ray findings is consistent with Tetralogy of Fallot? Egg-shaped heart Heart shaped like a snowman Increased pulmonary blood flow Lack of vascular congestion

Lack of vascular congestion The boy has central cyanosis with a harsh murmur characteristic of pulmonary stenosis. These findings are suspicious for tetralogy of Fallot (TOF). TOF is composed of four anatomic defects consisting of an overriding aorta, right ventricular hypertrophy, pulmonary stenosis, and ventricular septal defect (VSD). The clinical presentation depends upon the degree of pulmonary stenosis. The more severe the stenosis, the greater is the reduction of pulmonary blood flow and increased cyanosis. On examination, the patients are usually comfortable and in no distress. During hypercyanotic (tet) spells, patients usually become hyperpneic or agitated. The murmur of TOF is usually due to pulmonary stenosis and not the VSD. The murmur is due to the degree of obstruction and to the amount of flow across the obstruction. The diagnosis of TOF is generally made by echocardiography. Other tests that are often performed during the evaluation of TOF include chest radiography and electrocardiogram. The classic chest X-ray in TOF demonstrates a "boot-shaped" heart with an upturned apex and a concave main pulmonary artery segment. The heart size is often normal, and pulmonary flow will appear normal or decreased. Treatment of TOF involves surgical closure of the VSD as well as repair of the pulmonary stenosis. The timing of this procedure depends upon the degree of obstruction to pulmonary blood flow. The following are possible chest radiography findings in cyanotic heart diseases: egg-shaped heart (A) is found in transposition of the great arteries, heart shaped like a snowman (B) is described in total anomalous pulmonary venous return, and increased pulmonary blood flow (C) can be found in truncus arteriosus, transposition of the great arteries, and total anomalous pulmonary venous return.

0.2 seconds on an ECG is one small or one large block?

Large. Each small block is 0.04 seconds. Therefore, 5 small blocks to every 1 large block (5 x 0.04 = 0.2).

What lead is most common used for a rhythm strip?

Lead II.

Ischemic Heart Disease

Leading cause of death in the USA in both men and women • RFs: family hx, smoking, HTN, DM, cholesterol, male, age >55 • Atypical presentations occur in up to one third of patients • Chest pain, diaphoresis, nausea, hiccups, radiation to shoulder/jaw/back • Elderly, diabetics, females, history of stroke or heart failure: increased risk for atypical presentation • ECG: hyperacute T waves earliest sign of MI • Up to 50% of ECGs are normal or nonspecific • Troponin I most specific for cardiac injury • Nitroglycerin contraindicated in right ventricular MI and recent sildenafil use • Aspirin, nitrates, clopidogrel, heparin, and beta-blockers (within 24 hours) • Preferred treatment for AMI: angioplasty or stenting

Which of the following best describes the finding seen in the ECG above? Atrioventricular block Left bundle branch block Right bundle branch block Sinoatrial block

Left bundle branch block Bundle branch blocks are abnormal conduction abnormalities (not rhythm disturbances) in which the ventricles depolarize in sequence, rather than simultaneously, thus producing a wide QRS complex (> 120 msec) and a ST segment with a slope opposite that of the terminal half of the QRS complex. A left bundle branch block is a bifascicular block in which ventricular activation is by way of the right bundle branch. The impulse travels down the right bundle, activating the septum and the free wall of the right ventricle, and then continues on in the same direction to activate the free wall of the left ventricle. Because the dominant forces are traveling in the same direction, there is a tendency toward monophasic QRS complexes. The ECG in a LBBB will show a large wide R wave in lead I and a negative wave (QS or rS) in lead V1. AV block (A) produces PR interval lengthening, not R-R' complexes. The ECG in a RBBB (C) will show a wide S wave in lead I and a RSR' pattern in lead V1. Sinoatrial block (D) is represented electrocardiographically by a missed cycle or beat within otherwise normal appearing P waves and QRS complexes.

What is the most common cause of right heart failure?

Left heart failure.

Chronic Heart Failure Treatment

Lifestyle modifications • Diuretics: used for acute pulmonary edema, no mortality benefit • ACEIs: ↓ mortality in all classes • BBs: ↓ mortality in classes II, III, IV • Hydralazine with nitrates: ↓ mortality in African-Americans • Digoxin: used in refractory systolic dysfunction, no mortality benefit • Spironolactone: ↓ mortality in class III/IV • Isolated diastolic dysfunction: HR and BP control • Advanced treatments: mechanical assist device, ICD, heart transplantation

What infectious etiology is associated with complete heart block?

Lyme disease

Mean Arterial Pressure

MAP = (CO x SVR) + CVP • MAP = DBP + 1/3 (SBP-DBP) • Another way: MAP = [(2 x diastolic)+systolic] / 3

How do you calculate mean arterial pressure (MAP)? [DBP = diastolic blood pressure, SBP = systolic blood pressure] MAP = [DBP + (2 x SBP)]/3 MAP = DBP + 1/3(SBP-DBP) MAP = DBP + 2/3(SBP-DBP) MAP = SBP + 1/3(SBP-DBP)

MAP = DBP + 1/3(SBP-DBP) Calculation of the mean arterial pressure (MAP) provides a weighted average of the systolic blood pressure (SBP) and the diastolic blood pressure (DBP). It is a determination of tissue perfusion and is normally 70-100 mm Hg in adults. A MAP of approximately 60 is necessary to perfuse the coronary arteries, brain, and kidneys. MAP = [DBP + (2 x SBP)]/3 (A); MAP = DBP + 2/3(SBP-DBP) (C); and MAP = SBP + 1/3(SBP-DBP) (D) are incorrect formulas.

Asymptomatic Hypertension

MC causes of BP elevation: pain, anxiety • No workup indicated • Outpatient follow-up

A 54-year-old man with a history of schizophrenia presents to the ED after a syncopal episode. During your evaluation, he becomes diaphoretic and complains of dizziness. You are able to feel a radial pulse, and he is alert and talking with you. His rhythm strip is seen above. Which of the following represents the first-line treatment of this disorder? Amiodarone Labetalol Magnesium sulfate Synchronized cardioversion

Magnesium sulfate This ECG shows a rapid, irregular, wide-complex rhythm with multiple QRS morphologies or polymorphic ventricular tachycardia. This most commonly appears as a cyclical progressive change in cardiac axis—otherwise known as torsades de pointes. Torsades often occurs in the setting of a prolonged QT interval during sinus rhythm and is due to abnormal ventricular repolarization. The patient above has a history of schizophrenia. This condition is managed with antipsychotics (risperidone, olanzapine), which are associated with prolonging the QT interval. A QT interval of 500 msec (congenital or acquired) is a risk factor for development of torsades. In adults, however, most causes of QT prolongation are acquired and multifactorial involving drug interactions, myocardial ischemia, and electrolyte disturbances. The immediate management for a patient with torsades is IV magnesium sulfate given as a bolus. If this patient decompensates and loses his pulse, then he'll require defibrillation. Amiodarone (A) is an antidysrhythmic often used in patients with ventricular tachycardia and ventricular fibrillation. However, amiodarone itself causes QT prolongation and can be deleterious in patients with torsades. Labetalol (B) is a combination beta- and alpha-blocking agent that is contraindicated in ventricular dysrhythmias due to its AV-nodal effects. Administration during torsades will likely worsen the dysrhythmia. Although electrical cardioversion (D) is the standard treatment for many wide-complex tachycardias, torsades is less responsive to electricity. It is also difficult to synchronize due to the undulating axis of torsades. If the patient decompensates and loses his pulse, then defibrillation should be performed.

The presence of an implantable cardioverter-defibrillator is considered a strong relative contraindication for what diagnostic study?

Magnetic resonance imaging.

What is the most common X-ray finding in acute aortic dissection?

Mediastinal widening is seen in the majority of aortic dissection cases.

A 54-year-old woman presents with dyspnea on exertion. She states that it has been progressively worsening over the last few months and she is losing her ability to complete activities that she was able to do before without sitting down to "catch her breath." Cardiac auscultation over the apex reveals a low-pitched, diastolic murmur and an opening snap. Which of the following is the most likely diagnosis? Aortic regurgitation Aortic stenosis Mitral regurgitation Mitral stenosis

Mitral stenosis Mitral stenosis is best auscultated at the apex of the heart. The murmur is described as low-pitched and diastolic. An opening snap of the valve leaflets is heard at the beginning of S2 and can sometimes be confused with a splitting of the second heart sound. The sounds are diminished with inspiration and exaggerated with expiration. Mitral stenosis is most commonly caused by rheumatic fever in childhood but can also be congenital. Symptoms typically begin between ages 20 and 50 years and can include dyspnea on exertion, shortness of breath, and fatigue. Mitral stenosis is associated with atrial fibrillation which may be picked up on an electrocardiogram. Transthoracic echocardiography is used for diagnosis. This will demonstrate information regarding valve anatomy and blood flow dynamics. Doppler used during the echocardiogram can demonstrate concurrent atrial valve abnormalities as well as valve regurgitation. This information is used to stage the severity of stenosis. Stages are listed in severity from stage A to stage D with stage D being the most severe. Mitral regurgitation (C) and aortic stenosis (B) both present with systolic murmurs. Mitral regurgitation causes a mid-frequency holosystolic murmur and aortic stenosis presents with a crescendo-decrescendo systolic murmur. Aortic regurgitation (A) would best be heard near the base of the heart at the second right intercostal space. The murmur associated with chronic aortic regurgitation is an early diastolic murmur that is high-pitched and blowing in quality.

Which valvulopathy is commonly associated with premature ventricular contractions (PVCs)?

Mitral valve prolapse.

Ventricular Fibrillation

Most common cause: ischemic heart disease • ECG: irregular chaotic pattern without P waves or QRS complexes • Immediate defibrillation

What trio of physical exam findings might be present in a patient with Pericardial Tamponade?

Muffled Heart Sounds, JVD, Narrowing Pulse Pressure/Hypotension (Beck's Triad).

What is the most common etiology of ventricular fibrillation?

Myocardial ischemia.

Infantile dilated cardiomyopathy is most commonly caused by which etiology other than idiopathic causes?

Neuromuscular disorders such as Duchenne and Becker muscular dystrophy.

Patients with severe aortic regurgitation may benefit from which long-acting vasodilators?

Nifedipine XL or other long acting calcium channel blockers.

A patient presents with chest pain and the ECG seen above. Which of the following medications is contraindicated in this patient's management? Aspirin Clopidogrel Heparin Nitroglycerin

Nitroglycerin This patient presents with an inferior ST elevation myocardial infarction (STEMI) and the use of nitroglycerin is relatively contraindicated in management. In patients with myocardial ischemia or infarction, nitrates are used to decreased myocardial oxygen demand. They increase venous capacitance leading to decreased preload and are direct coronary artery vasodilators. Coronary artery vasodilation leads to increased blood flow to ischemic myocardium. The beneficial effects of nitrates are profound leading to their recommendation for most patients with a systolic blood pressure > 90 mm Hg. An inferior STEMI is one of these contraindications. Patients with an inferior STEMI may also have right ventricular infarct and be preload dependent. In a patient with an inferior STEMI, right ventricular infarct is suggested by the presence of ST elevation in lead III larger than that in lead II. A right ventricular infarct can be discovered by performing a right-sided ECG and looking for ST elevation in lead "RV4." In these patients, a preload reducing medication like nitroglycerin can lead to a precipitous drop in blood pressure. Aspirin (A) is the most beneficial treatment in patients with ACS and should be given to all patients unless they have a severe allergy. Clopidogrel (B) is an antiplatelet agent that is beneficial in patients that will be going to cardiac catheterization. Heparin (C) has also been found to be beneficial in STEMI and should only be withheld in patients with concomitant bleeding or a history of hypersensitivity reaction.

A 35-year-old woman is being evaluated at her annual well woman exam. She has no complaints and generally feels healthy. She has no past medical history and takes no medications. She walks 30 minutes five days per week with no dyspnea or discomfort. On physical exam her blood pressure is 118/68, pulse 64/min and respiration rate is 13/min. Her body mass index is 22. Cardiac auscultation reveals a 2/6 mid-systolic crescendo-decrescendo murmur heard best at the left lower sternal border without radiation. She has a normal S1 and S2 and normal cardiac impulse. Lungs are clear and peripheral pulses are normal. Electrocardiogram is normal. What is the next appropriate step in management? Cardiac magnetic resonance imaging No additional testing Transesophageal echocardiography Transthoracic echocardiography

No additional testing No additional testing is indicated in this patient. She most likely has a benign systolic ejection murmur. This is a common clinical situation and reassurance is appropriate. A systolic murmur is present in up to 60 % of patients, with 90 % being associated with a normal echocardiogram. A murmur is characterized by its intensity or grade, timing, configuration, frequency or pitch, and location. The most common causes of a mid-systolic murmur are innocent flow murmurs, an increase in flow rate across a normal semilunar valve, and aortic valve sclerosis. Innocent murmurs are typically systolic ejection murmurs located at the base of the heart, short and soft, grade 1/6 to 2/6, without radiation, with a normal S1 and S2, normal cardiac impulse, and no evidence of any hemodynamic abnormality. The benign characteristics of the murmur in this patient include a grade < 3/6, mid-systolic timing, lack of radiation, and the absence of additional abnormal heart sounds. The remainder of the physical examination and the electrocardiogram are normal, without any evidence of cardiac enlargement or dysfunction. Signs of more serious cardiac disease include an S4, grade >3/6 intensity, any diastolic murmur and fixed splitting of S2. Transthoracic echocardiography (D) is the primary test for diagnosis and assessment of valvular heart disease. The appropriateness of this test is guided by history and physical examination. It is indicated in symptomatic patients, in those with a systolic murmur grade 3/6 or greater, diastolic murmurs, continuous murmurs (begins after s1 and beyond s2), holosystolic murmurs, late systolic murmurs, murmurs associated with ejection clicks, or murmurs that radiate to the neck or back. This patient's murmur does not have any of these characteristics. Transesophageal echocardiography (C) may be useful in patients with poor imaging by transthoracic study or to evaluate the feasibility of surgical repair when surgery is planned. Cardiac magnetic resonance imaging (A) is indicated if both transthoracic and transesophageal echocardiograms are equivocal in a more suspicious murmur. It is good for the assessment of the aorta and cardiac chamber size and function.

A 52-year-old man presents from his primary care physician's office for evaluation of an abnormal electrocardiogram. His ECG is seen above. He has no symptoms. What is the appropriate intervention? Measurement of cardiac enzymes No intervention Telemetry observation Transcutaneous pacer pad placement

No intervention This ECG demonstrates an example of Type 1 second-degree atrioventricular (AV) block. AV block results from impaired conduction through some portion of the electrical circuit impairing communication between the atria and ventricles. In first and second degree AV block there is a partial disruption of the electrical circuitry as opposed to third degree heart block when there is no electrical communication between the atria and ventricles (AV dissociation). In second-degree heart block, some sinus impulses do not reach the ventricles at all. In type 1 second-degree AV block, there is lengthening of the PR interval until ultimately a beat is dropped. This is also known as Wenckebach or Mobitz I. In many cases, type 1 second-degree heart block is a normal variant. Other times, the etiology is likely related to increased vagal tone and in most cases requires no treatment. It can also occur in a myocardial infarction and usually resolves after the infarct period. Type 2 second-degree heart block is characterized by a dropped beat without any prolongation of the PR interval. Type 2 is never considered a normal variant. In the absence of any cardiac symptoms, the measurement of cardiac enzymes (A) in type 1 second-degree heart block is not necessary. Telemetry observation (C) is not required in type 1 second-degree heart block because it does not typically progress into other forms of heart block. In patients with type 2 heart block, this interruption of the electrical pathway occurs below the level of the AV node and can progress. Transcutaneous pacer pads (D) are often placed on the chest of a patient with complete heart block (type 3) because of the high chance for impaired cardiac output. These patients will require pacemaker placement if the heart block is not due to a reversible cause (e.g. hyperkalemia).

Name some medications which are prescribed for venous insufficiency?

No oral medications have been shown to be beneficial in the treatment of this condition.

Can a S4 be heard in normal individuals?

No.

Is antibiotic prophylaxis indicated in patients with isolated aortic stenosis?

No.

A 49-year-old man presents with chest pain. His medical history does not list any cardiac murmur, however, during examination, you hear a mitral regurgitant murmur. Blood pressure is equal in both the left and right arms. Although you have none to compare to, you order an electrocardiogram and notice ST segment depression in three different leads and T-wave inversion in two different leads. No other abnormalities are appreciated. A chest radiograph is read as normal. Initial lab testing shows an elevated troponin level. Which of the following is the most correct diagnosis? Aortic dissection Non ST-segment elevation myocardial infarction ST-segment elevation myocardial infarction Unstable angina

Non ST-segment elevation myocardial infarction Myocardial infarction encompasses both non-ST-segment elevation myocardial infarction (NSTEMI) and ST-segment elevation myocardial infarction (STEMI). It is defined as myocardial cell death and necrosis as diagnosed by a rise and fall in cardiac enzymes (in association with appropriate clinical presentation) or by pathologic findings of prior myocardial infarction (e.g. new Q waves on ECG). NSTEMI represents subtotal coronary thrombosis and myocardial ischemia infarct. Common symptoms include angina less than 30 minutes duration, dyspnea, diaphoresis and palpitations. Myocardial insult is also associated with a new-onset mitral regurgitation murmur, newly-auscultated S4 and a paradoxical S2. New infarcts can be identified with a positive troponin on laboratory testing. Aortic dissection (A) is associated with severe, tearing, knifelike pain that may radiate to the midscapular region. It is associated with asymmetric blood pressures in the arms, aortic regurgitation murmur, widened mediastinum on radiography and neurologic deficits. It is less likely than NSTEM in this scenario. STEMI (C) is associated with positive troponins and ST-segment elevation, not depression. Unstable angina (D) is not associated with positive troponin levels, but it can be associated with T wave inversion and ST depression.

What is a typical chest radiograph finding in acute mitral regurgitation?

Normal cardiac silhouette with severe pulmonary edema.

A 46-year-old woman is in your office for her yearly physical. What is your interpretation of her ECG? Atrial fibrillation Atrial flutter Normal sinus rhythm Sinus tachycardia

Normal sinus rhythm The ECG is representative of normal sinus rhythm with a rate of 80. Normal sinus rhythm has a rate between 60-100. The rhythm is regular with a 1:1 relationship of the P to QRS. The PR interval is 120-200 msec, the QRS complex is 60-100 msec. P waves are upright in leads I, II, and aVF. Atrial fibrillation (A) is an irregularly irregular rhythm due to uncoordinated atrial activation and random occurrence of ventricular depolarization. The atria are not contracting but they do discharge electrical impulses to the ventricles. However, no single impulse depolarizes the atria completely, so only an occasional impulse gets through the AV node. It is the most common sustained dysrhythmia in clinical practice. Atrial flutter (B) is a rapid atrial rhythm but due to nodal delay, ventricular response rate is slower. Therefore, atrial flutter always occurs with some sort of AV block so that not all impulses are conducted. The resulting block is often variable (2:1, 3:1, 4:1). P waves have characteristic sawtooth pattern. In sinus tachycardia (D) the SA node is the pacemaker that causes the atria to depolarize regularly and, thus, the ventricles to depolarize regularly. P waves and QRS complexes occur regularly and the rate is >100.

Premature Ventricular Contractions (PVCs)

Occur earlier than the next expected normal QRS • Wider than a normal QRS • QRS morphology is generally bizarre • No preceding P wave • Deflection of the ST segment and T wave is opposite that of the QRS • Followed by a compensatory pause

Which of the following should be assessed in the physical exam of a patient with essential hypertension? Costovertebral angle tenderness Deep tendon reflexes Ocular fundus Pupillary response

Ocular fundus Hypertension is defined as a systolic blood pressure of 140 to 159 mm Hg or a diastolic blood pressure of 90 to 99 mm Hg on two or more occasions. Hypertension is commonly seen in the primary care setting and risk factors include family history of hypertension, obesity, advancing age, African-American race, physical inactivity, high sodium diet, and diabetes. When taking the history of a patient with hypertension, questions should include duration of the diagnosis, previous treatment, and aggravating factors such as alcohol consumption, smoking, use of nonsteroidal anti-inflammatory agents and other prescription medications. The purpose of the physical exam is to assess for end-organ damage and cardiovascular disease as well as potential causes of secondary hypertension. Physical exam should include an accurate reading of blood pressure along with an assessment of general appearance, heart, lungs, neck, abdomen, and extremities. A fundoscopic exam of the eyes should be used to evaluate for hypertensive retinopathy including hemorrhage, papilledema and cotton wool spots. Costal vertebral angle tenderness (A) is assessed to rule out pyelonephritis or renal stones. Deep tendon reflexes (B) are evaluated in patients as part of a complete neurological exam. Neurological abnormalities seen in patients with hypertension include visual disturbance, focal weakness or confusion. Pupillary response (D) is assessed as part of cranial nerve testing in patients with neurological defects.

An elderly man complains of dizziness upon standing. He denies being dizzy once he has stood for 5 minutes, and also denies being dizzy when supine or seated. He denies associated chest pain, palpitations, or dyspnea. Which of the following tests should first be performed during the evaluation of this positional dizziness? Chest radiograph Head-up tilt-table testing Orthostatic vital signs Transesophageal echocardiography

Orthostatic vital signs Orthostatic hypotension is due to an inadequate physiologic response to postural changes. This condition mostly exists in the elderly population. It is estimated that nearly 25% of syncopal admissions to the ED are due to orthostatic hypotension. Symptoms of orthostatic hypotension include dizziness, weakness, fatigue, light-headedness, headache or syncope which occur after standing. Primary causes include frailty, dehydration, poor cardiac output or autonomic instability. However, there are a multitude of underlying conditions that can cause secondary orthostasis. These include anemia, hemorrhage, cardiac dysfunction, venous insufficiency, endocrine dysfunction (hypothyroidism, hypoaldosteronism, adrenal insufficiency, diabetes insipidus, hypokalemia) and neurologic dysfunction (autonomic neuropathy, vitamin B12 deficiency). If suspected, orthostatic vital signs must be obtained as follows: BP and heart rate must be measured in the supine position, then repeated after 3 minutes of standing. If normal, but orthostasis is still suspected, then the patient should be sent for head-up tilt-table testing. Head-up tilt-table testing (B) can help confirm a diagnosis of suspected orthostatic hypotension when orthostatic vital signs are nondiagnostic. It should be done after orthostatic vital signs are checked. A chest X-ray (A) and echocardiogram (D) may be used during the evaluation of orthostasis, but they would not be ordered initially. It is important first to consider orthostatic hypotension as the diagnosis, then once confirmed, begin evaluation of other causes with more specific testing.

Pericardial Tamponade

Patient will be complaining of dyspnea and chest pain PE will show muffled heart sounds, JVD, hypotension (Beck's triad), pulsus paradoxus ECG will show low voltage QRS, electrical alterans Echocardiography will show diastolic collapse of RV Treatment is pericardiocentesis

Hyperthyroidism

Patient will be complaining of heat intolerance, palpitations, weight loss, tachycardia, and anxiety PE will show hyperreflexia Labs will show low TSH and high free T4 Most commonly caused by Graves disease (autoimmune against TSH receptor) Treatment is methimazole or PTU Comments: Propylthiouracil (PTU) P for pregnant

Aortic Dissection

Patient will be older • With a history of HTN, smoking, Marfan syndrome • Complaining of sudden "ripping" or "tearing" CP radiating to back • PE will show asymmetric pulses/BP • CXR will show widened mediastinum • Diagnosis is made by CT or transesophageal echocardiogram (TEE) • Treatment is reduce BP, surgery

Aortic Stenosis

Patient will be older • With a history of diabetes, hypertension • Complaining of dyspnea, chest pain, syncope • PE will show crescendo-decrescendo systolic murmur that radiates to the carotids, paradoxically split S2, S4 gallop • Most commonly caused by degenerative calcification • Treatment is aortic valve replacement • Comments: murmur decreases with valsalva

Rheumatic Fever

Patient with a history of GAS infection • Complaining of fever, red skin lesions on the trunk and proximal extremities, and small, non-tender lumps located over the joints • PE will show JONES criteria: Joints, Oh, no carditis!, Nodules, Erythema marginatum, Sydenham's chorea • Labs will show anti-streptolysin O, anti-DNase B, positive throat culture, or positive rapid antigen test • Treatment is antibiotics, NSAIDs • Comments: Modified Jones Criteria for a first episode of acute rheumatic fever: need 2 major or 1 major and 2 minor

Tetralogy of Fallot

Patient with a history of episodes of cyanosis (tet spells) and squatting for relief • PE will show pulmonic stenosis, right ventricular hypertrophy, overriding aorta, VSD • CXR will show "boot-shaped" heart • Comments: Most common cyanotic congenital heart disease • Mnemonic: PROVe:: Pulmonic stenosis, Right ventricular hypertrophy, Overriding aorta, VSD

Mitral Regurgitation

Patient with a history of ischemic heart disease, endocarditis, MI, trauma • Complaining of dyspnea • PE will show blowing holosystolic mumur, best heard at apex with radiation to axilla, pulmonary edema, cardiogenic edema • Diagnosis is made by echo • Treatment is nitroprusside, dobutamine, intraaortic balloon pump, emergency surgery

Hyperkalemia

Patient with a history of renal failure, DKA, rhabdomyolysis, tumor lysis Complaining of lethargy, weakness, paralysis PE will show bradycardia, hypotension, cardiac dysrhythmia ECG will show peaked T waves, prolonged PR, wide QRS Treatment is calcium gluconate, insulin, albuterol, kayexalate, bicarbonate

Mitral Stenosis

Patient with a history of rheumatic heart disease Complaining of exertional dyspnea, hemoptysis PE will show loud S1, opening snap, low-pitched, rumbling diastolic apical murmur Most commonly caused by rheumatic heart disease Comment: Antibiotic prophylaxis for procedures prone to bacteremia

A patient is being evaluated for dyspnea and lower extremity swelling. On physical exam the patient has jugular venous distention with inspiration and 2 + pedal edema. Hepatojugular reflex is present. The patient has clear lung fields and no murmur is appreciated. Which of the following findings would support the diagnosis of constrictive pericarditis over restrictive cardiomyopathy? Cardiomegaly Left bundle branch block Pericardial knock S3 heart sound

Pericardial knock A Pericardial knock is associated with constrictive pericarditis, which is the most likely diagnosis in this patient. Approximately 50% of patients with constrictive pericarditis present with a pericardial knock, which is an accentuated heart sound occurring slightly earlier than a third heart sound. This would not be expected in restrictive cardiomyopathy. Conversely, an audible S3 is frequently present in persons with restrictive cardiomyopathy because of the abrupt cessation of the rapid ventricular filling. Constrictive pericarditis is a rare but disabling condition characterized by impaired filling resulting from restraint of ventricular diastolic expansion by a stiff pericardium. Any cause of pericarditis can lead to the condition. Some common causes include viruses, cardiac surgery, mediastinal irradiation and connective tissue disease. Patients present with dyspnea, fatigue and peripheral edema. Examination may show evidence of right sided heart failure including ascites, pedal edema, hepatojugular reflex and jugular vein engorgement with inspiration, also known as a Kussmaul sign. Pulmonary congestion is absent. Both constrictive pericarditis and restrictive cardiomyopathy can present with signs and symptoms of right sided heart failure making it difficult to distinguish one from the other. Further diagnostic testing and specific physical exam findings can aid in making the correct diagnosis. A S3 heart sound (D) is frequently present in persons with restrictive cardiomyopathy while an accentuated S2 or pericardial knock is associated with constrictive pericarditis. Cardiomegaly (A) on chest x-ray is more suggestive of restrictive cardiomyopathy whereas radiographic evidence of a calcified pericardium strongly suggests constrictive pericarditis. Depolarization abnormalities, such as a left bundle branch block (B) on electrocardiogram strongly favor restrictive cardiomyopathy.

A 76-year-old man with colorectal cancer presents to the ED with dyspnea and fatigue. He is hypotensive, tachycardic, tachypneic and afebrile. The jugular venous pulse rides high on lateral neck inspection. Cardiac sounds, but not breath sounds, are distant. There is no discernable friction rub or murmur, however, his blood pressure decreases during inspiration. An ECG reveals normal rhythm, increased rate and decreased voltages. Which of the following treatments is most appropriate in this patient's plan of care? Cardioversion Endarterectomy Hemodialysis Pericardiocentesis

Pericardiocentesis This scenario most likely represents cardiac tamponade. Pericardial tamponade refers to the dampening effect of rapidly accumulating pericardial effusion. An increase in intrapericaridal pressure compresses the heart chambers, decreases venous return and ultimately decreases cardiac output. As this occurs, it becomes ever more difficult for blood to flow from chamber to chamber. Causes include pericarditis, traumatic aortic dissection and myocardial rupture. Patients usually present with severe dyspnea, fatigue and hypotension. Typical exam findings include Beck's triad of hypotension, distant heart sounds and increased jugular venous pressure. Tachycardia and clear-sounding tachypnea are common. Pulsus paradoxus, a decrease in systolic blood pressure more than 10 mm Hg during inspiration, is also commonly present. However, pulsus paradoxus also accompanies constrictive pericarditis, congestive heart failure, pulmonary embolism, and end-stage obstructive pulmonary disease. Distant heart sounds and friction rubs may be present. Chest radiographs show large cardiac silhouettes, and ECGs may reveal a widespread decrease in voltage with an effusion and electrical alternans in tamponade. Classic echocardiographic findings are effusion, interventricular septal shift during inspiration, diastolic collapse of the right atrium and respiration-timed alterations in transvalvular flow. This medical emergency is treated with cardiopulmonary stabilization, pericardiocentesis (percutaneous drainage of pericardial fluid), cautious volume replacement and inotropic medications such as dobutamine. Cardioversion (A) is an electrophysiological treatment used to reverse arrhythmias, such as atrial fibrillation. This patient has a normal rhythm. Endarterectomy (B) is a surgical procedure used to remove atheromatous plaque from within a vessel. It is commonly used in the carotid arteries and aorta of patients with chronic atherosclerosis and distal ischemia, such as stroke symptoms or painful distal vasculopathy. This patient does not present with neurologic or peripheral arterial disease complaints. Hemodialysis (C) is used for patients with end-stage renal failure.

A 55-year-old man is being evaluated for sudden onset of chest pain. He describes the pain as sharp that is improved by sitting up and leaning forward. The chest pain is made worse by inspiration or coughing. An ECG shows new widespread ST segment elevation. Auscultation over the left sternal border is heard in the above audio clip. Which of the following is the most likely diagnosis? Aortic stenosis Mitral stenosis Patent ductus arteriosus Pericarditis

Pericarditis A pericardial friction rub is most commonly associated with pericarditis. It resembles the sound of squeaky leather and is often described as grating, scratching, or rasping. The pericardium is a double-walled sac around the heart. The inner (visceral) and outer (parietal) layers are normally lubricated by a small amount of pericardial fluid, but when inflammation of pericardium is present, the 2 walls move against each other with audible friction (the rub). In children, rheumatic fever is often the cause of pericardial friction rub. The friction rub is usually best heard between the apex and sternum but may be audible across the precordium. The murmur associated with aortic stenosis (A) is associated with an easily heard systolic, crescendo murmur loudest along the upper right sternal border at the 2nd right intercostal space radiating to the carotid arteries bilaterally. The murmur associated with mitral stenosis (B) is mid-diastolic and has a rumbling character. It is best heard with the bell of the stethoscope in the left ventricular impulse area with the patient in the lateral decubitus position. It usually starts with an opening snap. A patent ductus arteriosus (C) is classically associated with a machinery, crescendo/decrescendo murmur continuous from systole to diastole.

The ankle-brachial index is used to evaluate which disease?

Peripheral arterial disease (normal is 0.9-1.4; < 0.9 = PAD; >1.4 = calcified PAD).

What is the definitive treatment for sinus bradycardia?

Permanent pacemaker.

Name three genetic disorders, and their respective protein abnormalities, that are associated with aortic dissection?

Polycystic kidney disease (polycystin), Marfan's syndrome (fibrillin) and Ehlers-Danlos IV (type 3 procollagen).

What other disease do up to 50% of patients with temporal arteritis also have?

Polymyalgia rheumatica, which manifests as proximal muscle pain and stiffness, often involving the upper > lower extremities.

What is Dressler's syndrome?

Post-myocardial infarction pericarditis.

Hypertension: Eighth Joint National Committee (JNC 8) Recommendations

PreHTN: systolic blood pressure (SBP) 120-139 mmHg or diastolic blood pressure (DBP) 80-89 mmHg • Stage I HTN: SBP 140-159 mmHg or DBP 90-99 mmHg • Stage II HTN: SBP >160 mmHg or DBP >100 mmHg • Treatment goals: . >60 years: SBP <150, DBP <90 . All others: SBP <140, DBP <90 • 1st line rx for general population: thiazide, CCB, ACEI, or ARB • 1st line rx for African Americans: CCB or thiazide • Chronic kidney disease: Rx should include ACEI or ARB

A 78-year-old man complains of his "heart -stopping". After reviewing his ECG, you appreciate intermittent, large, wide QRS complexes which are not preceded by a P wave. Otherwise, the rhythm strip appears normal without PR interval, ST segment, P wave or T wave abnormalities. Which of the following is the most likely diagnosis? Atrial fibrillation Multifocal atrial tachycardia Non-ST-elevation myocardial infarction Premature ventricular contraction

Premature ventricular contraction Premature ventricular contractions (PVCs) appear as abnormal QRS complexes and T waves that occur in another underlying rhythm. PVCs have 6 characteristics: (1) Occur earlier than the next expected normal QRS; (2) Wider than a normal QRS; (3) QRS morphology is generally bizarre; (4) Preceding P wave is absent; (5) Deflection of the ST segment and T wave is opposite that of the QRS; (6) Followed by a compensatory pause. In general, PVCs are benign. However, when more than 3 occur simultaneously at a rate greater than 100, it is considered ventricular tachycardia. Atrial fibrillation (A) is diagnosed when disorganized electrical activity is seen in place of normal P waves. Apart from the intermittent oddly shaped QRS complex, the remainder of this patient's ECG shows normal P waves. Multifocal atrial tachycardia (B) is characterized by an irregular rhythm with rate > 100 and 3 or more P waves of variable morphology and PR interval variability. NSTEMI (C) is associated with T wave inversion and ST segment depression.

A 65-year-old man is brought to the ED complaining of nausea for the last two hours. On arrival to the emergency department, he has the cardiac rhythm seen above. His blood pressure is 110/70 mm Hg. He denies any headache, chest pain, or difficulty breathing. Which of the following is the most appropriate next step in management? Defibrillation Magnesium sulfate Procainamide Synchronized cardioversion

Procainamide The rhythm seen in the image is a monomorphic ventricular tachycardia. This occurs in most cases due to the presence of scar tissue in the myocardium. The first objective for treating any patient with a wide-complex tachycardia (WCT) is to evaluate for hemodynamic instability (hypotension, chest pain, dyspnea, altered mental status) because any patient with a WCT can deteriorate quickly as the rhythm degenerates into ventricular fibrillation. In stable patients, procainamide is the drug of choice and will terminate the rhythm in the vast majority of cases. It is superior to amiodarone and lidocaine. Not only is it useful in the termination of stable ventricular tachycardia, but it also blocks accessory pathway conduction, which terminates preexcited tachycardias. Sotalol, a class III antidysrhythmic with beta-blocker properties, can also be used in such patients though it is considered a second-line alternative. Defibrillation (A) is reserved for pulseless ventricular tachycardia and ventricular fibrillation. Magnesium sulfate (B) is the treatment of choice for polymorphic ventricular tachycardia (torsades de pointes). If medical management fails or the patient begins to decompensate, the patient should immediately receive synchronized cardioversion (D).

What drug is safe to administer to a patient with wide complex irregular tachycardia?

Procainamide.

Second Degree Heart Block, type I (Wenkebach/Mobitz I)

Progressive prolongation of PR interval until one QRS complex is dropped

Which of the following is a medical treatment for congenital long QT syndrome? Flecainide Propafenone Propranolol Quinidine

Propranolol Propranolol is the appropriate treatment for congenital long QT syndrome as beta blockers are the mainstay of treatment for this condition. Long QT syndrome is the most common channelopathy and is thought to affect 1 in 5000 persons. It is a disorder of myocardial repolarization characterized by a prolonged QT interval on ECG and an increased risk of sudden cardiac death: QTc is usually > 460 ms. This syndrome is associated with an increased risk of a characteristic life threatening polymorphic ventricular tachycardia known as torsades de pointes or "twisting of the points." The primary symptoms in patients with long QT syndrome include palpitations, syncope, seizures, and cardiac arrest, and patients usually have syncope related to polymorphic ventricular tachycardia. Factors conferring the highest risk for sudden cardiac death include a history of sudden cardiac arrest, recent syncope, and QTc interval greater than 500ms. Beta blockers, such as propranolol, have shown to reduce both syncope and sudden cardiac death. The goal of beta blocker therapy is to blunt the maximal heart rate achieved during exertion and are thought to interrupt the "trigger" for torsades de pointe and shorten the QT interval. Flecainide (A) and propafenone (B) are both class IC III antiarrhythmics which are commonly used for atrial and ventricular dysrhythmias; however, they can lengthen the QT interval. Quinidine (D) is a class IA antiarrhythmic that can also lengthen the QT interval.

Cor Pulmonale

Pulmonary HTN + RVH → right heart failure • Most common chronic cause: COPD • Most common acute cause: PE • Right heart catheterization

Which of the following is most closely associated with the development of acute cor pulmonale? Acute bronchitis Health care associated pneumonia Heart failure Pulmonary embolism

Pulmonary embolism Cor pulmonale is defined as an alteration in the structure and function of the right ventricle caused by a primary disorder of the respiratory system. Pulmonary hypertension is the common link between lung dysfunction and the heart in cor pulmonale. Cor pulmonale is usually a chronic condition, but acute pulmonary embolism (more common) and acute respiratory distress syndrome are associated with acute cor pulmonale. The underlying pathophysiology in massive pulmonary embolism causing cor pulmonale is the sudden increase in pulmonary resistance. In chronic cor pulmonale, RV hypertrophy (RVH) generally predominates. In acute cor pulmonale, right ventricular dilation mainly occurs. Acute bronchitis (A) is not associated with cor pulmonale. Heart failure (C) results from an abnormality in systolic or diastolic cardiac function. Chronically increased left heart pressures in poorly controlled heart failure can lead to cor pulmonale, but in most patients, this does not occur. Pneumonia (B) can increase cardiac demands but generally does not lead to pulmonary hypertension or right ventricular dilation.

Which of the following is the most common physical exam finding in an abdominal aortic aneurysm? Abdominal bruit Diminished femoral pulses Duodenal obstruction Pulsatile abdominal mass

Pulsatile abdominal mass A pulsatile abdominal mass is the most common physical examination finding in a patient with an abdominal aortic aneurysm (AAA). Patients with unruptured AAA rarely have any symptoms. If pain is present, it is usually gradual in onset and dull. Patients occasionally describe colicky pain that can be in the back or stomach making it easy to confuse this diagnosis with renal colic. The presence of acute severe pain typically heralds rupture. The most prominent physical examination feature is a pulsatile abdominal mass. Typically, the mass is palpated at the level of the umbilicus. The ability to palpate an AAA depends on the size of the AAA and the patients body habitus. An abdominal bruit (A) is an uncommon finding. Generally, patients have good distal perfusion and so femoral pulses will not be diminished (B). Duodenal obstruction (C) may occur but is very rare.

Inferior ST-Elevation Myocardial Infarction

RCA occlusion • ST elevations: II,III, aVF • RV infarction . ST elevations: V4R and V5R . JVD, hypotension . Rx: IVF • NTG contraindicated

Normal sinus rhythm

Rate 60-100 • Regular rhythm with P for every QRS • PR interval 120-200 • QRS 60-100

A 30-year-old woman with no past medical history presents to the emergency department complaining of substernal chest pain for two hours. It is not worse with exertion and was not relieved by sublingual nitroglycerin. She admits to some mild nausea. She does not smoke cigarettes or use any illicit drugs. Her family history includes a grandmother who died of a myocardial infarction at 84-years-old. Labs in the emergency department are unremarkable. Point of care troponin is negative and ECG reveals sinus rhythm. What is your next step in management? Dobutamine stress test Exercise stress test Reassurance Stress echocardiography

Reassurance Reassurance is most appropriate for this patient as her pretest probability for coronary heart disease it is very low. An initial estimate of the likelihood of coronary heart disease is referred to as the pretest probability and is based upon the clinical history. The pretest probability determines the need for noninvasive diagnostic tests or even coronary arteriography. The pretest probability is calculated using patient's age, sex and classification of their angina. Angina is classified as "typical," "atypical" or "nonanginal." Typical angina has three components: (1) sub-sternal pain or discomfort; (2) provoked by exertion or emotional stress; (3) relieved by rest or nitroglycerin or both. Atypical chest pain has two of the three components and nonanginal chest pain has one or none of the three components. In this clinical scenario, the patient is 30 years old, a woman, and has chest pain classified as nonanginal. This gives her a 2% likelihood of having coronary heart disease and stress testing is not indicated. Stress testing is typically needed for initial diagnosis and subsequent clinical management in patient with potential cardiac disease. The utility of cardiac stress testing is determined in part by pretest probability. In patients with low pretest probability, false positive results may lead to potential downstream testing and treatment that may cause more harm than good. Conversely, in patients with high coronary heart disease pretest probability, falsely negative studies lead to false security and missed diagnoses. Therefore, the greatest benefit is in patients with an intermediate pretest probability, in whom a positive test significantly increases the disease likelihood and a negative test significantly decreases the likelihood. Dobutamine stress test (A), exercise stress test (B) and stress echocardiography (D) are different forms of stress testing that have distinct indications including, but not limited to, the patient's ability to exercise, resting electrocardiogram, prior cardiac history, and structural heart status.

A 23-year-old man is admitted for a femur fracture. An admission ECG is shown above. What management is indicated? Atropine Beta blocker Reassurance Transvenous pacing

Reassurance The patient's ECG shows a first degree AV block; a normal variant finding in 2% of the population that requires no specific management. First degree AV block is defined as prolonged conduction of atrial impulses. The PR interval > 0.20 seconds (200 msec) and there is no loss of any atrial impulse. All impulses result in a ventricular response. Although it can be a normal variant, it can be seen in pathologic conditions like Lyme disease and should be followed up. However, no specific management is needed. Nodal depressing agents should be avoided or used with caution. Atropine (A) is required for patients with symptomatic bradycardia. Beta blockers (B) are AV nodal blockers and should be used with caution in patients with first degree AV blocks. Transvenous pacing (D) is required for higher degree AV blocks (second degree type II and third degree heart block).

Nitrate therapy works by which of the following mechanisms? Reducing afterload Reducing both preload and afterload Reducing cardiac contractility Reducing preload

Reducing both preload and afterload Nitrates work by reducing both preload and afterload. Nitroglycerine was the first treatment for angina pectoris and dates back to the 1800s. It still remains first-line drug therapy for many patients. Nitrates dilate veins and coronary arteries and to a lesser extent systemic arteries by relaxing vascular smooth muscle. Thus, nitrates reduce preload by increasing venous capacitance and improve coronary blood flow by coronary vasodilatation. Decreased preload lowers left ventricular end-diastolic pressure, thereby decreasing wall stress, resulting in a decrease in myocardial oxygen demand. At higher doses the afterload effects cause a drop in systemic blood pressure further decreasing wall stress and oxygen demand. Nitrates do not have a direct effect on cardiac chronotropy or inotropy. The primary adverse effects induced by nitrate therapy include hypotension, headache, and tachycardia. Nitrates should be avoided in patients with one or more of the following: systolic blood pressure less than 90 mm Hg, heart rate less than 50/min, or heart rate greater than 100/min. It should also be avoided in known or suspected right ventricular infarction, in patients who have taken a phosphodiesterase inhibitor for erectile dysfunction within the last 24 hours, in patients with hypertrophic cardiomyopathy or severe aortic stenosis. Nitrates do not work by only reducing afterload (A) or preload (D). Nitrates do not directly reduce cardiac contractility (C).

By what mechanism do ACE inhibitors cause hyperkalemia?

Reduction of aldosterone secretion.

A 62-year-old man with a history of sarcoidosis presents to his cardiologist with dyspnea, fatigue, and signs of right-sided heart failure. Physical exam shows elevated jugular venous pressure and hepatomegaly. Echocardiogram shows reduced diastolic filling. Magnetic resonance imaging shows gadolinium hyperenhancement. Which of the following is the most likely diagnosis? Dilated cardiomyopathy Hypertrophic cardiomyopathy Restrictive cardiomyopathy Takotsubo cardiomyopathy

Restrictive cardiomyopathy Restrictive cardiomyopathy occurs due to fibrosis or infiltration of the ventricular wall because of collagen-defect diseases. It can occur due to amyloidosis, endomyocardial fibrosis, hemochromatosis or sarcoidosis. Patients often present with decreased tolerance of exercise. Pulmonary hypertension is common. In advanced cases, patients develop right-sided congestive heart failure. Physical exam shows elevated jugular venous pressure (JVP) and Kussmaul's sign (a paradoxical rise in JVP on inspiration). Chest radiography shows mild to moderate cardiomegaly. Echocardiography is key to diagnosis, showing rapid, early diastolic filling and a small-normal sized, thickened left ventricle. MRI shows gadolinium hyperenhancement. Diuretics may be useful symptomatic treatment. Prognosis is poor if due to amyloidosis. Cardiac transplantation can be used if there is no systemic involvement Dilated cardiomyopathy (A) is often idiopathic or associated with longstanding alcohol abuse. Left ventricular dilation and cardiomegaly occur. Physical examination shows S3 and elevated jugular venous pressure. Hypertrophic cardiomyopathy (B) leads to an increased risk of sudden death in young athletes. Most patients are asymptomatic; others present with dyspnea on exertion, angina, or syncope. Takotsubo myopathy (D) occurs after a large discharge of catecholamines due to a major stressor. It is more common in postmenopausal women. Symptoms include angina and dyspnea.

Which common systemic diseases are associated with pericarditis?

Rheumatoid arthritis and connective tissue diseases.

Which of the following states a correct order of electrical current through the heart during one cycle of normal cardiac depolarization? Atrioventricular node -> sinoatrial node Bundle of His -> atrioventricular node Left bundle branch -> right bundle branch Right bundle branch -> purkinje fibers

Right bundle branch -> purkinje fibers A normal cycle of cardiac depolarization begins in the right atrium's sinoatrial node and passes through the right atrium's internodal tracts to activate the atrioventricular node. The AV node then sends the electrical impulse to the Bundle of His, which then passes current through the left and right bundle branches to the ventricular Purkinje fibers. Atrioventricular (AV) block is characterized as a delay in processing the electrical impulse within the atrioventricular node. This ultimately results in a delay in ventricular depolarization and contraction. There are three main types: first, second and third degree AV block. First-degree AV block is characterized as a prolonged PR interval > 0.2 sec, beginning at the start of the P wave and ending at the start of the QRS complex. Common causes of this type of block include electrolyte abnormalities, enhanced vagal tone (as in athletes), myocarditis or infarction and medications. Common medications which slow cardiac conduction through the AV node and produce a prolonged PR interval include beta and calcium-channel blockers, anticholinesterases and digitalis. Current passes normally from the SA to the AV node (A). Normal conduction is from the AV node to the Bundle of His (B), not the reverse. The left bundle does not normally propagate current to the right bundle (C).

What vessel is responsible for the majority of inferior myocardial infarctions?

Right coronary artery (80%) with the remainder due to occlusion of a dominant left circumflex artery.

What is the Sokolow-Lyon electrocardiographic definition of left ventricular hypertrophy?

S wave in V1 + R wave in V5 or V6 ≥ 35 mm.

What is the abnormal heart sound heard in the above audio recording? Holosystolic murmur S3 S4 Systolic ejection murmur

S3 The S3 heart sound, also referred to as a protodiastolic gallop or ventricular gallop, is generally associated with acute heart failure. The sound is associated with early diastolic filling and is heard in such pathologic states as volume overload and left ventricular systolic dysfunction. It occurs at the beginning of diastole and produces a rhythm classically compared to the cadence of the word Kentucky (S1 = Ken; S2 = tuck; S3 = y). The S3 may be normal in people under 40 years of age, may sometimes be heard in pregnancy, and in some athletes but should disappear before middle age. S3 is a dull, low-pitched sound best heard with the bell placed over the cardiac apex with the patient lying in the left lateral decubitus position. Holosystolic murmurs (A) start at S1 and extend up to S2. They are usually due to regurgitation such as in mitral regurgitation, tricuspid regurgitation, or ventricular septal defect. S4 (C) occurs just after atrial contraction and immediately before the systolic S1. It produces a rhythm classically compared to the cadence of the word Tennessee (S4 = Tenn; S1 = ess; S2 = ee). S4 is caused by the atria contracting forcefully in an effort to overcome a hypertrophic ventricle. Systolic murmurs (D) are classified as ejection or regurgitant murmurs. Ejection murmurs emanate from the pulmonic or aortic valves or their surrounding structures. There are many causes of systolic ejection murmurs, including valvular aortic stenosis, atrial septal defect, acute mitral regurgitation, and ventricular septal defect.

Cardiac Electrical Conduction System

SA node → AV node → bundle of his → bundle branches → purkinje fibers

A 34-year-old man presents to the emergency department with complaints of worsening chest pain, fever, and malaise. The pain is pleuritic, worsens when he lies down and improves when he leans forward. On exam, he appears unwell, but is not in acute distress. Auscultation over the precordium reveals a scratchy, grating sound with a normal S1 and S2. Which of the following is the most likely electrocardiogram finding in this patient? Shortened PR interval and slurring of the QRS complex ST segment depression in leads V2-V4 ST segment elevation with reciprocal ST depression in leads aVR and V1 U waves and flat T waves

ST segment elevation with reciprocal ST depression in leads aVR and V1 This patient with pericarditis will most likely have diffuse ST segment elevation with reciprocal ST depression in leads aVR and V1. Pericarditis is an inflammation of the pericardial sac. The pericardium is composed of a visceral layer, which forms the epicardium, and the parietal layer. These layers are separated by a space that normally contains 15-50 mL of serous fluid. During acute pericarditis, changes in the ECG reflect inflammation of the epicardium, since the parietal layer of the pericardium is electrically inert. The ECG evolves through as many as four stages during acute pericarditis. Lasting hours to days, stage 1 involves generalized ST segment elevations and PR segment depression. ST and PR segments normalize during stage 2. Stage 3 development is variable in some patients. Stage 3 is characterized by diffuse T wave inversions. Stage 4 represents normalization of the ECG. However, not all causes of pericarditis cause alterations in the ECG. Uremic pericarditis rarely causes epicardium inflammation and therefore, the ECG will show none of the changes described above. Chest pain is the most consistent finding associated with pericarditis. The pain is typically described as sharp and pleuritic. The tendency for the pain to decrease in intensity when the patient leans forward is a distinctive characteristic. Sitting up and leaning forward reduces the pressure on the parietal pericardium and allows for diaphragm splinting. In addition to an ECG, echocardiography should be performed to evaluate for pericardial effusions and to rule out cardiac tamponade. Unless contraindicated, nonsteroidal anti-inflammatory drugs (NSAIDs) are considered first line treatment for acute pericarditis. NSAIDs should not be used to treat pericarditis after myocardial infarction or uremic pericarditis. The typical ECG of Wolff-Parkinson-White syndrome ECG has a shortened PR segment and slurring of the initial QRS segment (A). Wolff-Parkinson-White syndrome is a congenital condition, characterized by abnormal conduction pathways between the atria and ventricles. ST segment depression in leads V2-V4 (B) is characteristic of myocardial ischemia. While both myocardial infarction and pericarditis can cause ST segment elevation, pericarditis differs from myocardial infarction in several ways. The morphology of the ST segment in pericarditis usually remains concave. The ST segment seen with an MI is typically convex. Additionally, ST segments in MI are limited to the corresponding affected artery, and reciprocal changes are usually noted in opposite leads. Pericarditis produces generalized ST segment elevation. Q waves may also be noted in MI. Pericarditis does not result in Q waves unless there is concomitant myocarditis. U waves and flattened T waves (D) are caused by hypokalemia. In severe hypokalemia, the T wave becomes completely flat. Prolonged vomiting and diarrhea, eating disorders, drugs, hyperaldosteronism, and renal disease are potential causes of hypokalemia.

Atrial Flutter

Sawtooth pattern Atrial rate: 250-300/minute Ventricular rate: 150- +/-30 AV node conducts every 2 or 3 atrial impulses

List some causes of left bundle branch block?

Senile fibrosis of the conduction system, chronic hypertension, chronic cardiac ischemia, chronic congestive heart disease and valvular disease.

Which of the following is correct regarding cardiac markers in regards to myocardial infarction? Creatinine phosphokinase (CK) is specific to cardiac tissue Myocardial infarction can be ruled out with a single serum myoglobin Serum troponin is more sensitive than creatinine phosphokinase (CK-MB) Troponin levels return to normal in 2-3 days

Serum troponin is more sensitive than creatinine phosphokinase (CK-MB) Serum troponin is a more sensitive marker of myocardial infarction than creatinine phosphokinase (CK-MB). Biochemical markers (troponin, CK-MB, myoglobin) are essential in the diagnosis and risk stratification of myocardial infarction. Troponin I and T are myocardium specific proteins that are released from myocardial cells after cell damage occurs. Both have been found to be highly specific and sensitive for the early detection of myocardial injury and have supplanted the use of other biomarkers. The presence of serially negative troponins also predicts a low risk for an event. Troponin levels may not be detectable for up to 12 hours after symptoms onset in some patients with acute myocardial infarction. However, with highly sensitive troponin assays, detection may be possible as early as 3 hours. CK (A) is a protein that is found in large quantities in cardiac and skeletal muscle and is thus nonspecific. Myoglobin (B) is found in all muscle tissue and rises earlier in myocardial infarction than troponin. However, a single normal level does not rule out a myocardial infarction. While CK levels return to normal within 48-72 hours, troponin levels take up to 5-14 days (D) to return to baseline.

Which of the following cardiac auscultation findings is most consistent with mitral valve regurgitation? Harsh, crescendo-decrescendo systolic murmur Loud S1, opening snap, and a diastolic rumble Mid-systolic click and late systolic murmur Soft S1 and a loud, blowing holosystolic murmur

Soft S1 and a loud, blowing holosystolic murmur A soft S1 and a loud, blowing holosystolic murmur are typical findings of mitral valve regurgitation. Mitral regurgitation is defined as abnormal reversal of blood flow from the left ventricle back into the left atrium. This results in an increased preload and reduced afterload. In chronic MR, the left atrium and ventricle dilate to maintain stroke volume. Mitral regurgitation (MR) is a common cardiac valvular disorder due to a primary abnormality of the valve apparatus or secondary to another cardiac disease. In the developed world, degenerative causes and coronary heart disease are the most common etiologies of MR. The most common presenting symptoms of mild to moderate, chronic MR are exertional dyspnea, fatigue, and paroxysmal to persistent atrial fibrillation. Severe, acute MR can present with heart failure and pulmonary edema. Physical exam findings may include a diminished S1, a pansystolic murmur that radiates to the axilla, and a hyperdynamic left ventricular impulse. The murmur of MR typically increases with leg raise, decreases with Valsalva maneuver, and has little respiratory variation. The findings of MR on chest radiography may include cardiomegaly and straightening of the left heart border. Electrocardiography, while not required for MR diagnosis, can be used to evaluate for concurrent conditions, such as left ventricular hypertrophy and atrial fibrillation. The diagnosis of MR is confirmed with transthoracic echocardiography (TTE). TTE is also used to determine the severity of regurgitation and for identifying the cause of MR. Medical therapy involves afterload reducing agents, diuretics, and beta blockers. Anticoagulation may be required for patients with atrial fibrillation. Surgical repair is indicated for patients with moderate to severe MR. All patients with MR should be referred to a cardiologist. A harsh crescendo-decrescendo murmur (A) that radiates to the neck is typical of aortic stenosis. Aortic stenosis can be caused by a congenital bicuspid valve or calcification of the valve. Angina pectoris, symptoms of heart failure, and syncope are the classic triad of aortic stenosis symptoms. The murmur of mitral stenosis is characterized by a loud S1, opening snap, and a diastolic rumble (B). Rheumatic disease is a common cause of mitral stenosis. Symptoms of mitral stenosis include exertional dyspnea, orthopnea, and paroxysmal nocturnal dyspnea. A mid-systolic click and late systolic murmur (C) is characteristic of mitral valve prolapse. Mitral valve prolapse is commonly seen in healthy, young women. The majority of patients with mitral valve prolapse have some degree of mitral regurgitation. Mitral valve prolapse is generally asymptomatic, but can be associated with palpitations, dyspnea, fatigue, atypical chest pain, and anxiety.

What is the most common bacteria responsible for infective endocarditis?

Staphylococcus aureus.

What organism is classically associated with infectious endocarditis seen in patients with colon cancer?

Streptococcus bovis.

Hypertrophic cardiomyopathy in a young athlete can commonly lead to which disastrous result?

Sudden cardiac arrest and death.

On which side of the valve is the vegetation typically located in infectious endocarditis?

Superior.

What treatment can be curative in constrictive pericarditis?

Surgical pericardectomy.

What is the equation for the target heart rate used for an exercise stress test?

Target heat rate = 85 % (220 - age (in years)).

Vasculitis

Temporal arteritis: PMR, carotid artery branches affected, vision loss, Rx: immediate steroids • Takayasu's arteritis: Asian, decreased pulses • PAN: generalized without lung involvement, HBV • Buerger's disease: smokers, claudication of hands/feet • Granulomatosis with polyangiitis (GPA): Upper and lower respiratory sx + renal sx, c-anca • Microscopic polyangitis: similar to GPA but without nasopharyngeal involvement, p-ANCA • Churg-Strauss syndrome: vasculitis + eosinophilia + asthma • Cryoglobulinemia: HCV, malaise, skin lesions, joint pain • Behçet's disease: oral and genital ulcers, hyperreactivity to needle sticks

What is the most common cardiac cause for cyanosis of children (of any age)?

Tetralogy of Fallot.

An S3 extra heart sound is heard best with which end of a stethoscope?

The bell.

What is the name of the accessory bundle in WPW syndrome?

The bundle of Kent.

What is the significance of an elevated creatinine kinase- MB and normal troponin value?

The injury is likely due to release from non-cardiac tissue.

Patients with aortic dissection may present with hoarseness due to compression of which nerve?

The recurrent laryngeal nerve.

Which coronary artery most commonly supplies the inferior wall with blood?

The right coronary artery.

What blood pressure defines a hypertensive emergency?

There is no defining blood pressure; all that is needed is an elevated pressure in the setting of acute end-organ damage.

What is the most commonly used first line agent to treat hypertension?

Thiazide diuretics, such as hydrochlorothiazide or chlorthalidone.

What class of diabetes medications are contraindicated in the setting of heart failure?

Thiazolidinediones, such as pioglitazone and rosiglitazone. Metformin should also be avoided in patients with decompensated heart failure due to the risk of lactic acidosis.

What is the classic echocardiographic finding in constrictive pericarditis?

Thickened pericardium.

Which pericarditis cases are most prone to develop tamponade?

Those which are idiopathic or due to malignancy or uremia.

Aspirin is the first-line antiplatelet used in managing stable angina in all cases except for which patients?

Those with a recent MI or coronary stent, in which case, clopidogrel is the recommended first-line antiplatelet agent.

What dysrhythmia are patients with prolonged QT interval at risk for?

Torsades de pointes.

A 24-year-old woman with no past medical history presents with left wrist pain after a fall. The left extremity is grossly deformed and the patient complains of severe pain. The patient has a blood pressure of 183/100 mm Hg. While awaiting X-rays, what management is indicated for the patients elevated blood pressure? Arrange admission for blood pressure control Start an oral beta-blocker and monitor for response Start intravenous beta-blocker and admit to the intensive care unit Treat the patient's pain and reassess the blood pressure

Treat the patient's pain and reassess the blood pressure The patient presents with a markedly elevated blood pressure in the setting of pain from a trauma and should have pain control initiated and her blood pressure rechecked. Hypertension is defined as a persistent SBP >140 mm Hg or DBP >90 mm Hg. Pain and anxiety are common causes of elevated blood pressure and heart rate in the outpatient setting. Historically, patients with elevated blood pressure and nonspecific symptoms were referred to as hypertensive urgency but this term has fallen out of use. In a patient presenting with elevated blood pressure who does not have signs or symptoms of end-organ damage, the clinician's focus should be on identifying external reasons for the elevated pressure and treating or addressing these. In this case, the reduction or relief of pain will likely lead to decreased blood pressure. Patients with elevated blood pressure and an absence of end-organ damage (e.g. acute coronary syndrome, aortic dissection, encephalopathy, change in renal function) do not require admission (A) for management. A primary care physician in the outpatient setting best manages these patients. Starting a beta-blocker (B) will not be beneficial in a patient with acute pain as the cause of elevated blood pressure. Similarly, administration of an intravenous beta-blocker and admission to the intensive care unit (C) is not indicated as the patient exhibits no end-organ damage.

A 40-year-old woman, who actively uses intravenous drugs, presents to the ED with fever and fatigue for the past 3 days. In the ED, her vital signs are BP 126/82, HR 90, RR 16, oxygen saturation 99% on room air, and temperature 101.6°F. On exam, a murmur is noted. A transesophageal echocardiography is ordered for suspected endocarditis. Which of the following valves is most likely to be affected? Aortic Mitral Pulmonic Tricuspid

Tricuspid This patient most likely has bacterial endocarditis. Endocarditis is more common in patients with valvular abnormalities, prosthetic valves, and IV drug users. Common pathogens include Staphylococcus aureus and viridans group streptococci. Of note, viridans streptococcus endocarditis commonly presents after dental work. Endocarditis presents most commonly with fever and malaise, although other signs and symptoms may be present. Although the classic triad is fever, anemia, and a heart murmur, this rarely presents clinically. IV drug users usually have right-sided endocarditis, most commonly affecting the tricuspid valve. The murmur noted on exam is usually tricuspid regurgitation. Labs should be ordered for those with suspected endocarditis, as one would expect leukocytosis, anemia, increased ESR, and increased CRP. Three blood cultures from three separate venipuncture sites should be taken, with the first and last draw occurring at least one hour apart. The most useful diagnostic imaging is an echocardiogram, especially a TEE, demonstrating vegetations. The Duke criteria are the most widely used and accepted criteria to clinically diagnose bacterial endocarditis. Empiric antibiotics should be initiated in those with suspected or confirmed endocarditis once cultures have been drawn, with antibiotics being altered as needed once the pathogen and susceptibility return. Left side valves (aortic, mitral) are most commonly involved in endocarditis outside of intravenous drug abusers and prosthetic valves. The mitral valve (B) is the most commonly affected valve in infective endocarditis, although the aortic valve (A) may also be affected. The pulmonic valve (C) is much less commonly affected.

You discover a blowing, holosystolic murmur in a newborn boy, heard loudest at the left sternal border. A pediatric cardiologist diagnoses Ebstein's anomaly. In addition to a malformed right atrium and ventricle, which of the following abnormalities would you most expect to see on this patient's echocardiogram? Left ventricular hypertrophy Overriding aorta Tricuspid insufficiency Tricuspid stenosis

Tricuspid insufficiency Tricuspid regurgitation (or incompetence, or insufficiency) manifests as a blowing, pansystolic murmur. It is commonly associated with a thrill. It is most intense in the left, fourth intercostal space, however, it can radiate to the apex, making it difficult to differentiate from a mitral regurgitation murmur. Like tricuspid stenosis, it is quite rare, affecting only 1% of the US population. Causes include rheumatic heart disease, right ventricular dilation, myxomatous degeneration and varied connective tissue disorders. It is part of Ebstein's anomaly, a congenital heart defect in which the tricuspid leaflets attach to the right ventricular wall, leading to a larger than normal right atrium and smaller than normal right ventricle. Ebstein's anomaly is also commonly associated with an atrial septal defect, patent foramen ovale and the pre-excitation, re-entrant conduction defect of Wolff-Parkinson-White syndrome. Diuretics are the mainstay of treatment, and valvuloplasty or valve repair, if necessary, is far more common than valve replacement. Left ventricular hypertrophy (A) is not associated with Ebstein's anomaly. It is commonly due to chronic systemic hypertension. An overriding aorta (B) is one of the four congenital cardiac malformations of Tetralogy of Fallot, not Ebstein's anomaly. Tricuspid stenosis (D) does not produce a systolic murmur.

Which of the following murmurs is associated with an increase in right atrial pressure? Aortic regurgitation Mitral regurgitation Mitral stenosis Tricuspid regurgitation

Tricuspid regurgitation Valve disorders are often characterized in terms of stenosis (incomplete opening of the valve, thereby increasing the resistance in blood flow) or regurgitation (an incomplete closure of a valve, resulting in a backflow of blood). The tricuspid valve is 1 of 2 atrioventricular (AV) valves and lies between the right atrium and right ventricle. In tricuspid regurgitation, there is incomplete forward flow from the right atrium into the right ventricle and blood flows from the right ventricle into the right atrium during systole. This backflow of blood results in increased right atrial pressure. Tricuspid regurgitation is caused by right ventricular dilation secondary to pulmonary hypertension, rheumatic heart disease, and infective endocarditis. Patients may complain of fatigue and dyspnea on exertion. On exam, there may be a holosystolic murmur best heard at the xiphoid area adjacent to the left sternal border. The aortic valve is one of 2 semilunar valves (the other being the pulmonary valve) and is positioned between the left ventricle and aortic trunk. Aortic regurgitation (A) results in a backflow of blood and an increase in pressure of left ventricle during diastole. The mitral valve is the 2nd AV valve and is situated between the left atria and left ventricle. Mitral regurgitation (B) causes a backflow into the left atria during systole, increasing left atrial pressure. In mitral stenosis (C), the mitral valve cannot fully open during diastole, resulting in an increase in resistance in blood flow through the valve and increasing the left atrial pressure while decreasing the filling of the left ventricle.

Cardiac Biomarkers

Troponin Highest sensitivity and specificity Time detectable from onset: 3-12 hours Peak: 24-48 hours Return to baseline: 5-14 days CK-MB Time detectable from onset: 3-12 hours Peak: 24 hours Return to baseline: 48-72 hrs Useful for dx of reinfarction Myoglobin First to appear, first to peak, first to decline Lacks specificity

Which antibiotic is first line for endocarditis prophylaxis prior to a dental procedure?

Two grams of Amoxicillin 1 hour prior to procedure.

A man who presents with syncope is placed on the cardiac monitor. On the monitor you note a repeating trend of 6 P waves, 5 of which are followed by a narrow QRS complex and 1 of which is not followed by a QRS complex. The PR interval during this trend progressively increases. Which of the following is the most likely diagnosis? First-degree AV block Third-degree AV block Type I second-degree AV block Type II second-degree AV block

Type I second-degree AV block A key distinction between first-degree and second-degree heart block is that in first-degree block the P wave is always followed by a QRS complex. In other words, the ratio of P waves to QRS complexes is 1:1, or, the electrical signal from the atria always passes to the ventricles. In second-degree AV block, the electrical impulse sometimes gets to the ventricles. There are two main types of second-degree AV block. In Mobitz type I, or Wenckebach, second-degree block, there is a progressive beat-to-beat lengthening of the PR interval until a P wave does not conduct through the AV node. The absent conduction and resultant "missing" QRS complex is called a "dropped" QRS, which represents an absent beat of ventricular contraction. First-degree AV block (A) has a 1:1 ratio of P waves to QRS complexes. Mobitz type II second-degree heart block (D) is characterized by a nonconducted P wave which is not preceded by progressive PR interval prolongation. AV dissociation, or third-degree AV block (B), occurs when none of the P waves conduct through the AV node. This complete AV block occurs with separate atrial and ventricular rates. There is no discrete correlation or trend between P waves and QRS complexes.

Which is more serious, type I (Mobitz I/Wenckebach) or type II (Mobitz II) second-degree AV block?

Type II, which is more associated with complete heart block and cardiac arrest.

How long after cessation of alcohol intake do alcohol withdrawal seizures occur?

Typically, they occur between 6 and 48 hours after stopping alcohol intake but they can occur up to 5 days later.

Deep Vein Thrombosis

Unilateral leg swelling • Phlegmasia cerulean dolens (painful blue leg) = massive iliofemoral thrombosis with venous insufficiency • Phlegmasia alba dolens (painful white leg) = massive iliofemoral thrombosis → arterial spasm • Risk stratification: Well's criteria • Modality of choice: ultrasound • Treatment: . ​Proximal DVT: heparin, warfarin . Massive DVT: thrombectomy . Isolated calf vein thrombosis: aspirin, ultrasound in 2 - 5 days . Recurrent DVT on warfarin: heparin, IVC filter . Propagation of DVT on warfarin + heparin: IVC filter

A detailed history and examination does not aid in the evaluation of new-onset hypertension in a 47-year-old man. In an attempt to search for an underlying cause, you order a basic metabolic panel, complete blood count, lipid panel and ECG. Which of the following tests should also be added to this standard diagnostic screen of secondary hypertension? Aortic ultrasound Echocardiogram Renal angiogram Urinalysis

Urinalysis The evaluation of hypertension involves specific history questions, exam findings and laboratory testing. A standard history should include questions about the presence of comorbidities (coronary artery disease, atherosclerosis, congestive heart failure, previous myocardial infarction, peripheral arterial disease, hypercholesterolemia, transient ischemic attacks and strokes, diabetes, renal insufficiency, endocrinopathies, retinal disease, connective tissue disease and obstructive sleep apnea), medications (oral contraceptives, corticosteroids, NSAIDs and cyclosporine), social history (salt intake, tobacco use, alcohol use, cocaine and methamphetamine use, dietary intake, and exercise trends) and family history of hypertension. The physical examination must include serial bilateral arm BP measurements, cardiac exam (murmurs, evidence of left ventricular hypertrophy), peripheral vascular and skin exam (edema, bruits, capillary refill, striae, moon fascies), thyroid exam, abdominal exam (masses, bruits), fundoscopic exam and neurologic exam. Baseline laboratory-diagnostic testing should include a basic metabolic panel, complete blood count, urinalysis, lipid panel and ECG. Aortic coarctation or aneurysm are rare causes of secondary hypertension. They are associated with abdominal, cardiac and peripheral vascular findings, which are missing in the above patient. Ultrasound (A) is the diagnostic test of choice when evaluating such abnormalities. Echocardiography (B) may be warranted if the clinician suspects left ventricular hypertrophy, valvulopathy, coronary artery disease or prior myocardial infarction as a cause of secondary hypertension, all of which would be suggested by abnormalities found during history and physical examination. Renal angiogram (C) offers great sensitivity and specificity in determining renovascular disease as a cause of secondary hypertension. However, it is not considered part of an initial evaluation, and should be considered if the initial tests above are noncontributory.

A 46-year-old woman with a past history of a DVT was recently diagnosed with Burkitt's lymphoma. Recent blood work revealed a creatinine of 2.3. She is currently hospitalized to receive chemotherapy when she suddenly develops tachycardia to a rate of 130 and oxygen saturation of 91%. Which of the following is the most appropriate test to confirm the diagnosis of pulmonary embolism? Chest X-ray CT angiogram of the chest with intravenous contrast D-dimer V/Q scan

V/Q scan According to the modified Wells Criteria this patient has a high clinical probability for a pulmonary embolism (PE). This patient has multiple risk factors for a PE including cancer, previous DVT, and immobilization secondary to hospitalization. She also has a heart rate greater than 100 and a decreased oxygen saturation, which are associated with PE. The elevated creatinine is a contraindication to receiving intravenous contrast and therefore a she cannot undergo CT angiography, which is the usual test of choice. Therefore, the most appropriate test for this patient is a V/Q scan. CT angiogram of the chest (B) is the preferred diagnostic modality in patients with suspected PE. However, the test requires that IV contrast is administered, which is contraindicated in patients with elevated creatinine. D-dimer (C) is useful in risk stratification of patients suspected of having a PE. A negative D-dimer rules out PE in patients with low or moderate risk established from Well's scoring. A chest X-ray (A) has poor sensitivity for PE.

A 46-year-old woman presents to the Emergency Department with fever, cough, and hemoptysis. She has a history of intravenous opioid use. Vital signs are BP 110/65 mm Hg, HR 120 beats per minute, RR 20 breaths per minute, and T 103.4°F. On auscultation of the chest, you hear a faint systolic ejection murmur. Which of the following is the most appropriate initial therapy? Ampicillin and gentamicin Ceftriaxone Oxacillin and rifampin Vancomycin and ceftriaxone

Vancomycin and ceftriaxone This patient is presenting with signs and symptoms of infectious endocarditis. Risk factors for infectious endocarditis include rheumatic heart disease, congenital or acquired valvular disease, and intravenous drug use. Right-sided endocarditis involves either the pulmonic or tricuspid valve. It is classically seen in intravenous drug users. Organisms implicated in right-sided endocarditis include Staphylococcus aureus, Streptococcus pneumoniae, and gram negative bacteria. Presenting symptoms often include fever, cough, hemoptysis, chest pain, and dyspnea. Right-sided endocarditis is frequently misdiagnosed initially as pneumonia. Management includes antibiotics for the suspected organism based on the clinical situation. In an IV drug user, coverage should include methicillin-resistant Staphylococcus aureus, staphylococci, streptococci and enterocci. The most appropriate antibiotic choice for this patient would be vancomycin and ceftriaxone. Ampicillin and gentamicin (A) would be an appropriate antibiotic regimen for resistant Streptococcus viridans and non-resistant enterococci. It does not adequately cover MRSA, which should be covered. Ceftriaxone (B) does not cover the typical infecting organisms. Oxacillin and rifampin (C) would be partially appropriate for a patient with prosthetic valve endocarditis as it covers for Staphylococcus aureus and adds rifampin to penetrate the biofilm on prosthetic valves; however, gentamicin should be added in order to cover for Pseudomonas aeruginosa. This would also not be an adequate regimen for MRSA as oxacillin would need to be substituted with vancomycin.

A 76-year-old man with a history of hypertension presents after a syncopal event. He also reports decreased exercise tolerance over the last two months. He is currently asymptomatic. His ECG is normal sinus rhythm with no changes. On examination, he has a harsh systolic murmur that radiates to the carotid arteries. Which of the following is true about the most likely cause of his syncope? Critical disease is defined by heart valve area less than 2 cm2 It typically results from rupture of the chordae tendinae The murmur increases with valsalva Vasodilators and diuretics should be used with caution

Vasodilators and diuretics should be used with caution Aortic stenosis is an abnormality of the heart valve that prevents left ventricular outflow. This results in left ventricular hypertrophy which eventually impairs diastolic filling and results in increased myocardial oxygen demand. The most common cause is degenerative calcification associated with diabetes, hypertension, and advanced age. Exam findings include a harsh systolic murmur that radiates to the carotids as well as an S4 gallop. Carotid pulses may be delayed and diminished and a narrowed pulse pressure is typically present. Classic symptoms start with dyspnea and chest pain and progress to syncope and congestive heart failure. Although patients may be asymptomatic for a long period of time, once symptoms develop mortality increases significantly. Because patients with critical aortic stenosis are preload dependent, vasodilators and diuretics should be avoided as they can cause significant hypotension. Definitive treatment is aortic valve replacement. A normal aortic valve area is greater than 3 cm2. Critical aortic stenosis (A) is defined by a heart valve area of less than 0.8 cm2. Mitral regurgitation is most commonly caused by post-myocardial infarction rupture of the chordae tendinae (B) or papillary muscle dysfunction and presents with acute onset of pulmonary edema and midsystolic apical murmur. The murmur of aortic stenosis decreases with valsalva (C).

An 81-year-old man with 10 years of coronary artery disease presents with chest pain and altered mental status. His ECG shows consecutive, large and wide QRS complexes. P waves cannot be appreciated. His pulse is 188 bpm. Which of the following is the most likely diagnosis? Atrial fibrillation Atrial flutter Ventricular fibrillation Ventricular tachycardia

Ventricular tachycardia Ventricular tachycardia is described electrocardiographically as "continuous PVCs", or more specifically, consecutive, fast, large and wide QRS complexes. A wide QRS complex clues one in that the underlying electrical problem is in the ventricles. Ventricular tachycardia can be further defined as monomorphic (QRSs are the same form) versus polymorphic (differing appearance of the QRSs), or sustained (>30 seconds) versus non-sustained (<30 seconds, self-terminates). In this tachydysrhythmia, the ventricles depolarize and contract so fast that cardiac output decreases, especially in the elderly or those with diseased myocardium or coronaries. This perpetuates ventricular ischemia leading to a precipitous decline if the dysrhythmia is not treated. Atrial fibrillation (A) and atrial flutter (B) both have narrow QRS complexes, either irregular or regular respectively. Ventricular fibrilation (C) is identified by a totally erratic appearance of unidentifiable waves. Discrete QRS complexes are absent.

A 60-year-old man presents with the ECG seen above. His blood pressure is 80/60 mm Hg. Which of the following is the most likely diagnosis? Atrial flutter Atrioventricular reentry tachycardia Supraventricular tachycardia Ventricular tachycardia

Ventricular tachycardia Ventricular tachycardia (VT) is present when there are >3 consecutive premature ventricular contractions occurring at a rate > 100. P waves are usually absent and the QRS complexes are wide (> 120 msec) and may be bizarre appearing. Ventricular tachycardia is classified as "monomorphic" (QRS complexes look the same) or "polymorphic" (QRS complexes have varying morphology). Sometimes it is difficult to distinguish ventricular tachycardia from supraventricular tachycardia (SVT) with aberrancy (presence of a bundle branch block). Nonetheless, any patient with a wide complex tachycardia who is hemodynamically unstable should undergo immediate electrical cardioversion. Atrioventricular reentry tachycardia (A) is a type of SVT. It is more commonly associated with regular pulse and monomorphic narrow QRS complexes. Atrial flutter (B) is more commonly associated with regular tachycardia and sawtooth appearance of P waves. It is sometimes difficult to distinguish ventricular tachycardia from supraventricular tachycardia with aberrancy. Any patient with a wide complex tachycardia who is hemodynamically unstable should be treated as having ventricular tachycardia. Some electrocardiographic characteristics that support VT over SVT (C) are extreme left axis deviation, QRS concordance (all QRS complexes from V1 to V6 are either all positive or all negative), and fusion or capture beats, which indicates AV dissociation.

The presence of coronary disease or infarcted myocardium is more closely associated with supraventricular tachycardia or ventricular tachycardia?

Ventricular tachycardia.

A 68-year-old man is being managed on a multi-drug antihypertensive regimen for essential hypertension. His blood pressure is at goal, but he notes persistent constipation and bilateral pedal edema. Which of the following medications is most likely responsible for these side effects? Enalapril Losartan Metoprolol Verapamil

Verapamil Verapamil, along with most other calcium channel blockers, is known to cause pedal edema and constipation, particularly in elderly patients. Other adverse effects of calcium channel blockers include headaches, flushing, and bradycardia, especially with the dihydropyridine calcium channel blockers (such as nifedipine, amlodipine, or nicardipine). Edema development can usually be lessened by adding an ACE inhibitor or angiotensin receptor blocker to the patient's regimen. Calcium channel blockers are generally well tolerated and valuable components of an antihypertensive regimen that work by causing peripheral vasodilation while minimizing the reflex tachycardia common with other vasodilating agents. Hypertension is diagnosed when a patient's systolic blood pressure exceeds 140 mm Hg, or their diastolic blood pressure exceeds 90 mm Hg on two separate occasions. If a work-up for secondary causes of hypertension proves fruitless, a patient is presumed to have essential hypertension and should be managed with diet, exercise, and medications to reach their goal blood pressure (acceptable goals range from <120 - <140/<90 depending on patient circumstances). Proper control is vital to reducing complications such as cardiovascular disease, cerebrovascular disease, hypertensive nephropathy, and hypertensive retinopathy. A medication regimen must be tailored for each patient, but will generally include some combination of diuretics, ACE-inhibitors, angiotensin receptor blockers, beta-blockers, calcium channel blockers, or centrally-acting sympatholytics. Enalapril (A), an ACE-inhibitor, is a common initial medication for treating hypertension. However, it is more likely to cause a chronic, dry cough and hyperkalemia rather than pedal edema or constipation. Losartan (B) is an angiotensin receptor blocker. This class of antihypertensive medications has similar utility to an ACE-inhibitor, and it also is likely to contribute to the development of hyperkalemia. It is unlikely to cause the edema and constipation seen with calcium channel blockers. Metoprolol (C), a beta-blocker, is also used for managing hypertension by decreasing the heart rate and cardiac output. Beta-blocker side effects normally include fatigue, lethargy, bronchospasm, and bradycardia, but do not commonly cause constipation or pedal edema.

A 50-year-old woman with a history of mitral stenosis secondary to rheumatic fever presents with atrial fibrillation. She does not have a history of heart failure, hypertension, diabetes mellitus or previous stroke or transient ischemic attack. What is the most appropriate management for this patient? Aspirin Dabigatran No anticoagulation Warfarin

Warfarin Warfarin is the most appropriate management. This patient has mitral stenosis and requires anticoagulation with warfarin regardless of CHA2DS2 - VASc score. Validated risk factors for thromboembolism in patients with atrial fibrillation include mitral stenosis, previous thromboembolism, heart failure, systolic dysfunction, diabetes, hypertension, presence of a mechanical heart valve and older age. In patients without significant valvular disease, the mostly commonly used method to determine choice of thromboprophylaxis is the CHA2DS2 - VASc score. Those with a score of 0 are at low risk and anticoagulation is a clinical decision. A score of 1 is low to moderate risk and antiplatelet or anticoagulation therapy should be considered. A score of 2 or more is considered moderate to high risk and anticoagulation is recommended. Patients with mitral stenosis who have chronic atrial fibrillation have a stroke risk that may be as high as 7-15 % per year. It is recommended that patients with mitral stenosis and atrial fibrillation be started on warfarin, and it should be continued indefinitely to decrease the risk of systemic thromboembolism. The goal INR is between 2.0 to 3.0. Aspirin (A) is not appropriate because the presence of mitral stenosis necessitates anticoagulation with warfarin. No anticoagulation (C) is not appropriate as this patient has a risk factor that require further therapy. Newer oral anticoagulants, such as dabigatran (B), are now approved for prevention of systemic embolism in adults with nonvalvular atrial fibrillation. However, the use of warfarin in patients with mitral stenosis is recommended because patients with significant mitral valve disease were excluded from the trials of these newer agents.

What formula is used to calculate the average weight in a child?

Weight (kg) = (2 x age in years) + 8.

What is antidromic conduction?

When the impulse is conducted anterograde through the accessory pathway and retrograde through the AV node. The QRS complex is wide.

Ventricular tachycardia is most difficult to distinguish from which other dysrhythmia?

Wide-complex supraventricular tachycardia (SVT with aberrancy such as a bundle-branch block).

A 37-year-old man presents to the emergency department with chest pain and shortness of breath. His medical history is significant for uncontrolled type I diabetes and end-stage renal disease on hemodialysis. His last dialysis was four days ago and he missed his appointment this morning. His labs are notable for a fingerstick blood glucose 300 mg/dL, potassium 7.0 mmol/L, magnesium 2.0 mEq/L, and phosphorus 4.0 mmol/L. Which of the following findings is most likely to be seen on this patient's ECG? Osborn waves QT interval prolongation U waves Widened QRS complex

Widened QRS complex Symptomatic hyperkalemia is a life-threatening electrolyte abnormality typically seen in patients with underlying acute or chronic kidney disease. It can also be seen in conditions that cause increased tissue breakdown such as tumor lysis syndrome, rhabdomyolysis, and crush injuries. Muscle weakness and paralysis, cardiac conduction abnormalities and cardiac dysrhythmias are the most serious manifestations of hyperkalemia. Symmetrical peaked T waves with a shortened QT interval and ST-T segment depression are the earliest ECG changes seen in patients with hyperkalemia. Worsening hyperkalemia results in progressive lengthening of the PR interval and QRS duration, disappearance of the P wave, and ultimately widening of the QRS complex into a sinusoidal pattern. Treatment of hyperkalemia includes antagonizing the membrane effects of potassium, driving extracellular potassium into cells, and removing potassium from the body. Osborn waves (A), also known as J-waves, are positive deflections in the junction of the QRS complex and the ST segment. They are most commonly seen in patients with hypothermia. QT interval prolongation (B) is seen in hypokalemia, hypomagnesemia, and hypocalcemia. QT interval shortening is seen in hyperkalemia. U waves (C) are small deflections that follow the T wave and are typically seen in hypokalemia, rather than hyperkalemia.

Which of the following is most commonly seen on chest radiography in a patient with an aortic dissection? Abnormal aortic contour Calcification of the aorta Pleural effusion Widened mediastinum

Widened mediastinum The chest radiograph can be normal in 10%-20% of patients with known aortic dissection. Therefore, it cannot be used to rule out the diagnosis. In cases where there is an abnormality, a widened mediastinum is most commonly seen, occurring in approximately 62% of cases, according to the International Registry for Aortic Dissection (IRAD) study. A tortuous aorta, common in hypertensive patients, may be hard to distinguish from a widened mediastinum. In the elderly, a mildly or moderately enlarged (i.e., tortuous) aorta can be a normal finding. However, a markedly enlarged aorta is abnormal. In younger individuals (age <40 years), chest radiography is potentially more useful because any degree of aortic dilation is abnormal and indicates an increased risk of aortic dissection. Other radiologic abnormalities seen on chest radiography include a pleural effusion, left apical cap, tracheal deviation to the right, depression of left main stem bronchus, esophageal deviation, and loss of the paratracheal stripe. The IRAD study revealed an abnormal aortic contour (A) in 50% of cases, calcification of the aorta in 60% of cases (B), and pleural effusion (C) in 19% of cases.

Which of the following diseases is characterized by the presence of a delta wave on an electrocardiogram? Charcot-Marie-Tooth disease Parkinsonism Parkinson's disease Wolff-Parkinson-White syndrome

Wolff-Parkinson-White syndrome Wolff-Parkinson-White (WPW) syndrome is a congenital cardiac condition in which there is an abnormal conduction pathway (accessory bundle) between the atria and ventricles other than the atrioventricular node. This pathway provides the basis for a reentrant circuit which bypasses the typical delay thru the AV node, typically resulting in premature ventricular depolarization and supraventricular tachyarrhythmia. The premature depolarization of some of the ventricle produces a "slurring" of the normal depolarization wave thru the AV node. This is seen as a gradual upsloping of the QR segment, called a delta wave, which creates an illusion of PR interval shortening and QRS lengthening. Charcot-Marie-Tooth disease (A) is a hereditary sensorimotor neuropathy which commonly leads to peroneal muscular atrophy. Parkinson's disease (C) is a degenerative disorder of the dopamine-generating cells in the substantia nigra, which causes parkinsonism symptoms. Parkinsonism (B) is a term used to describe the typical parkinsonian symptoms which arise from a variety of diseases, with Parkinson's Disease being one of these causes. Parkinsonism generally refers to tremor, rigidity, slowed movements and balance impairment.

Which of the following is associated with a shortened PR interval? Brugada Syndrome Mobitz 2° AV Block Wellens Syndrome Wolff-Parkinson-White syndrome

Wolff-Parkinson-White syndrome Wolff-Parkinson-White syndrome (WPW) is a congenital condition in which there is an abnormal accessory conduction pathway called the bundle of Kent. This accessory conduction pathway causes a premature depolarization of the ventricles by bypassing the AV node. The AV node typically acts to delay depolarization (the PR interval) to the ventricles to allow them to fill before contracting (QRS complex). The accessory pathway causes early depolarization of the ventricles resulting in an early QRS complex, thereby shortening the PR interval. On an ECG, this is represented by an upslope of the QRS complex (delta wave) that causes a widened QRS complex (>120 msec). The primary significance of WPW syndrome is that it predisposes patients to the development of tachydysrhythmias, particularly atrial fibrillation. Brugada Syndrome (A) is a hereditary condition characterized by a right bundle branch block-like pattern with ST elevation in Leads V1 to V3. Wellens Syndrome (C) is described by a large inverted T wave in Leads V2 and V3 and is associated with critical stenosis of the proximal left anterior descending coronary artery. Both Brugada and Wellens syndromes have normal PR intervals. Mobitz 2° AV Block (B) occurs at the level of AV node. Depolarizations from the atria are blocked and do not reach the ventricles, resulting in some P waves not being followed by a QRS complex (thereby, having no PR interval). The P waves that are followed by QRS complexes have normal PR intervals.

Heart Block: Second Degree Type I (Wenckebach/Mobitz I)

• Progressive PR interval prolongation until QRS dropped • Block within AV node • Usually benign

ACE Inhibitors

• Use: HTN, DM II • MOA: block the conversion of angiotensin I to angiotensin II • ADR: cough, angioedema • Comments: Names end in "pril"


Set pelajaran terkait

Cognitive Psychology Exam 2 (Chapters 5-8)

View Set

The first Emperor of china, Ch.22

View Set

Chapter 62: Management of Patients with Burn Injury

View Set

Les verbes suivants de la page 24 (No et moi)

View Set

US History - American Revolution

View Set

Fahmy 2017 Spanish - = book - = T R A F A L G A R

View Set